Step 2 Clinical Knowledge (CK)
SAMPLE TEST QUESTIONS
A Joint Program of the Federation of State Medical Boards of the
United States, Inc., and National Board of Medical Examiners
T
his booklet was updated July 2023.
For Public Release
Copyright © 2023 by the Federation of State Medical Boards of the United States, Inc. (FSMB), and the National Board of Medical
Examiners
®
(NBME
®
). All rights reserved. Printed in the United States of America. The United States Medical Licensing
Examination
®
(USMLE
®
) is a joint program of the FSMB and NBME.
USMLE Step 2 CK Test Question Formats …………………………………...…………..………. 3
Introduction to USMLE Step 2 CK Sample Test Questions …………………………..….. 9
USMLE Laboratory Values ………………………………...………..…….….………..……………... 10
USMLE Step 2 CK Sample Test Questions……………………………………………..…............ 13
Block 1, Items 1–40 …………………………………………………………………..….......... 13
Block 2, Items 4180 ……………………………………………..….............................. 29
Block 3, Items 81120 ……………………………………………..…........................... 47
Answer Form for USMLE Step 2 CK Sample Test Questions………………………..…... 63
Answer Key for USMLE Step 2 CK Sample Test Questions………………………….……. 64
CONTENTS
3
The following are strategies for answering one-best-answer items:
Read each question carefully. It is important to understand what is being asked.
Try to generate an answer and then look for it in the response option list.
Alternatively, read each response option carefully, eliminating those that are clearly incorrect.
Of the remaining response options, select the one that is most correct.
If unsure about an answer, it is better to guess since unanswered questions are automatically counted as wron
g
an
swers.
Patient Scenario Formats
Patient scenarios for any Single Item or Sequential Item Set may be provided in either Vignette (paragraph) format, or in
Chart/Tabular format. Test items using the chart/tabular format are designed to resemble patient charts but are not
intended to be an exact representation of a patient chart. Questions written in Chart/Tabular format will contain
relevant patient information in list form, organized in clearly marked sections for ease of review. Familiar medical
abbreviations may be used within Chart/Tabular format questions.
Single-Item Questions
A single patient-centered vignette is associated with one question followed by four or more response options. The
response options are lettered (ie, A, B, C, D, E). A portion of the questions involves interpretation of graphic or pictorial
materials. You are required to select the best answer to the question. Other options may be partially correct, but there is
only ONE BEST answer. This is the traditional, most frequently used multiple-choice question format on the examination.
Example Question 1
A 32-year-old woman with type 1 diabetes mellitus has had progressive renal failure over the past 2 years. She has not yet
started dialysis. Examination shows no abnormalities. Her hemoglobin concentration is 9 g/dL, hematocrit is 28%, and
mean corpuscular volume is 94 μm
3
. A blood smear shows normochromic, normocytic cells. Which of the following is the
most likely cause?
(
A) Acute blood loss
(B) Chronic lymphocytic leukemia
(C) Erythrocyte enzyme deficiency
(D) Erythropoietin deficiency
(E) Immunohemolysis
(F) Microangiopathic hemolysis
(G) Polycythemia vera
(H) Sickle cell disease
(I
) Side
roblastic anemia
(J
) β-
Thalassemia trait
(
Answer: D)
USMLE STEP 2 CK TEST QUESTION FORMATS
4
Example Question 2
Patient Information
Age: 18 years
Gender: F, self-identified
Ethnicity: unspecified
Site of Care: emergency department
History
Chief Complaint: "My roommate took a lot of pills and said she wanted to die."
History of Present Illness:
• p
atient brought by ambulance 2 hours after a suspected acetaminophen overdose
• r
oommate had returned to their dorm and found the patient distraught; patient was holding an empty bottle of
acetaminophen and said, "I just want to die."
• roommate estimates patient consumed 20 to 30 tablets of 500-mg acetaminophen
Past Medical History:
• ge
neralized anxiety disorder
• ma
jor depressive disorder
Medications:
• citalopram
Allergies:
• n
o known drug allergies
Family History:
• n
oncontributory
Psychosocial History:
• c
ollege freshman; parents live 8 hours away
Physical Examination
Temp
Pulse
Resp
BP
O
2
Sat
Wt
BMI
36.8°C
89/min
16/min
108/59 mm Hg
99%
63 kg
22 kg/m
2
(98.2°F)
on RA
(140 lb)
• A
ppearance: awake and alert but distraught and fearful; patient says, "This was totally stupid."
• P
ulmonary: clear lung fields
• C
ardiac: regular rhythm; S
1
and S
2
, with an early systolic murmur
• A
bdominal: soft and nontender; liver and spleen cannot be palpated
N
eurologic: no abnormalities; fully oriented
Q
uestion: In addition to obtaining serum acetaminophen concentration, which of the following is the most
appropriate next step in management?
(
A) Administer N-acetylcysteine
(B) Arrange hemodialysis
(C) Gastric lavage
(D) Initiate sodium bicarbonate infusion
(
Answer: A)
5
Sequential Item Sets
A
single patient-centered vignette may be associated with two or three consecutive questions about the information
presented. Each question is associated with the initial patient vignette but is testing a different point. You are required to
select the ONE BEST answer to each question. These questions are designed to be answered in sequential order. You must
click “Proceed to Next Item” to view the next item in the set; once you click on this button, the next question will be
displayed, and you will not be able to change the answer to the previous question.
Example Sequential Question
A 35-year-old woman is brought to the emergency department because of worsening pain and swelling of her right knee
for the past 2 days. She has been taking acetaminophen for the knee pain during the past 2 days, but the pain is worse
today. She has not had any trauma to the knee or any previous problems with her joints. She is otherwise healthy and she
currently takes an oral contraceptive. She is sexually active and has a 10-year-old son who lives with her. She is a
receptionist at a local hotel and she tells you she must stand often while working. She is 160 cm (5 ft 3 in) tall and weighs
52 kg (115 lb); BMI is 20 kg/m
2
. Temperature is 37.9°C (98.9°F). The right knee is erythematous, swollen, and tender; there
is pain on movement. No other joints are affected. X-ray of the knee shows an effusion but no structural abnormalities of
the joint.
W
hich of the following is the most appropriate next step in diagnosis?
(
A) Arthrocentesis of the knee
(B) Blood cultures
(C) Complete blood count
(D) MRI of the knee
(E) Urine cultures
(Answer: A)
Ar
throcentesis is done. The synovial fluid is cloudy. Gram stain is negative. Analysis of the synovial fluid shows a
leukocyte count of 120,000/mm
3
and 90% neutrophils. Which of the following is the most appropriate additional
test on the synovial fluid?
(
A) Culture for bacteria
(B) Glucose measurement
(C) Polarized light microscopy
(D) Protein level
(Answer: A)
6
Abstract Set Format
The abstract item format includes a summary of an experiment or clinical investigation presented in a manner commonly
encountered by a physician, eg, as an abstract that accompanies a research report in a medical journal. Examinees must
interpret the abstract in order to answer questions on various topics, including
Decisions about care of an individual patient
Biostatistics/epidemiology
Pharmacology/therapeutics
Use of diagnostic studies
Example Abstract Set
Question
In
children with type 1 diabetes mellitus, what factors are associated with increased risk for microalbuminuria and
macroalbuminuria?
Methods
D
esign: Inception cohort followed for a mean of 9.8 years.
Setting: St. Bartholomew’s Oxford Diabetes Register, Oxford, England, UK.
Patients: 527 children < 16 years of age (mean age 9 years) who were diagnosed with type 1 diabetes mellitus and
included in the Diabetes Register from 1986 to 1997 (90% follow-up).
Prognostic factors: Mean glycated hemoglobin concentration (HbA
1c
), female sex, mean blood pressure, history of
smoking, and age at diagnosis.
Outcomes: Microalbuminuria (albumin-to-creatinine ratio 3.5 to 35 mg/mmol in boys and 4.0 to 47 mg/mmol in girls, in
two annual, consecutive, early morning urine samples), and macroalbuminuria (albumin-to-creatinine ratio > 35
mg/mmol in boys and > 47 mg/mmol in girls).
Ma
in results
135 p
atients (26%) developed microalbuminuria, with a cumulative prevalence of 25.7% (95% CI 21 to 30) after 10 years
and 51% (CI 41 to 61) after 19 years. Of the 135 patients with microalbuminuria, there was a cumulative prevalence of
regression to the normoalbuminuric range of 52% (CI 42 to 62) after 4.9 years from the onset of microalbuminuria. 18
patients (3%) developed macroalbuminuria, with a cumulative prevalence of 14% (CI 13 to 15) after 3.2 years from onset
of microalbuminuria. HbA
1c
and female sex were associated with increased risk for microalbuminuria (Table). HbA
1c
and
persistent and intermittent microalbuminuria were associated with increased risk for macroalbuminuria (Table).
C
onclusions
In children with type 1 diabetes mellitus, poor glycemic control and female sex were associated with development of
microalbuminuria. About half of patients with microalbuminuria regressed to normoalbuminuria. Poor glycemic control
and persistent or intermittent microalbuminuria were associated with development of macroalbuminuria.
S
ources of funding: Diabetes UK; Juvenile Diabetes Research Foundation; Wellcome Trust; NIHR Cambridge Biomedical
Research Centre.
Structured abstract based on: Amin R, Widmer B, Prevost AT, et al. Risk of microalbuminuria and progression to macroalbuminuria
in a cohort with childhood onset type 1 diabetes: prospective observational study. BMJ. 2008;336:697-701. 18349042
Abstract continues on next page.
7
Factors associated with renal outcomes in children with type 1 diabetes mellitus*
Outcomes
Prognostic factors
At a mean 9.8 years of follow-
up
Hazard ratio (95% CI)
Microalbuminuria
HbA
1c
(per % increase)
1.4 (1.3 to 1.5)
Female sex
1.4 (1.02 to 2.1)
Diastolic blood pressure
1.0 (1.0 to 1.0)
Systolic blood pressure
1.0 (1.0 to 1.0)
History of smoking
1.3 (0.9 to 2.0)
Younger age at diagnosis
1.0 (1.0 to 1.1)
Macroalbuminuria
HbA
1c
(per % increase)
1.4 (1.2 to 1.8)
Persistent microalbuminuria†
28 (8.0 to 96)
Intermittent microalbuminuria‡
8.8 (2.4 to 31)
Female sex
1.3 (0.5 to 3.3)
Diastolic blood pressure
1.1 (0.9 to 1.1)
Systolic blood pressure
1.1 (0.9 to 1.1)
History of smoking
1.3 (0.4 to 4.1)
Younger age at diagnosis
1.0 (1.0 to 1.1)
*HbA
1c
= glycated haemoglobin.
†Presence of microalbuminuria at every annual visit after first detection.
‡Positive microalbuminuria followed by regression to normoalbuminuria, then recurrence to microalbuminuria at a later date.
A
12-year-old girl is brought to the office for a follow-up examination 6 months after being diagnosed with type 1
diabetes mellitus. The patient feels well. She says she smokes cigarettes occasionally when with friends. She is not
sexually active. She is at the 60th percentile for BMI. Her temperature is 37.0°C (98.6°F), pulse is 80/min, respirations
are 16/min, and blood pressure is 108/72 mm Hg. Physical examination shows no other abnormalities. Results of
urinalysis are within the reference ranges. Her father, who has type 1 diabetes mellitus and chronic renal disease,
asks what markers will be followed to determine his daughter's risk for developing chronic renal disease. Which of
the following patient characteristics most increases her risk for developing microalbuminuria over the next 10 years?
(
A)
A
ge at diagnosis
(B) BMI
(C)
Di
astolic blood pressure
(D)
G
ender
(E
) S
moking history
Answer: D
W
hich of the following current or potential future findings is most likely to increase the patient’s risk for developing
macroalbuminuria?
(A) Age at diagnosis
(B)
C
igarette use
(C)
E
levated systolic blood pressure
(D)
G
ender
(E
) P
oor glycemic control
A
nswer: E
Abstract set continues on next page.
8
O
ne year later, the patient's urinalysis shows microalbuminuria for the first time. In a population of children of a
similar age who have type 1 diabetes mellitus and had microalbuminuria detected for the first time, which of the
following outcomes is most likely?
(A) Hemoglobin A
1c
will normalize within 10 years
(B)
Hype
rtension will be diagnosed within 2 years
(C)
M
acroalbuminuria will be detected within 1 year
(D)
M
icroalbuminuria will resolve within 5 years
Answer: D
N
OTE: When additional question formats are added to the examination, notice will be provided on the USMLE website
(www.usmle.org). You must monitor the website to stay informed about the types of questions that occur in the
examination, and you must practice with the downloadable sample test questions available on the USMLE website in
order to be fully prepared for the examination.
9
The following pages include 120 sample test questions. Most of these practice questions are the same as those on the
USMLE website.
O
NLINE:
Please note that reviewing the sample questions as they appear on pages 1362 is not a substitute for practicing on the
website.
1. You should run the tutorial and items in the interactive testing experience on the USMLE website well before your
test date.
2. The items in the interactive testing experience include additional questions and formats that do not appear in this
booklet, such as items with associated audio.
CO
NTENT:
You should become familiar with all test question formats that will be used in the actual examination.
A
lthough the sample questions exemplify content on the examination, they may not reflect the content coverage on
individual examinations.
In
the actual examination, questions will be presented in random order. The questions will be presented one at a time in
a format designed for easy on-screen reading, including a Normal Laboratory Values button (Table included here on pages
1012). Photographs, charts, and x-rays in this booklet are not of the same quality as the pictorials used in the actual
examination. In addition, you will be able to adjust the brightness and contrast of the computer screen.
T
IMING:
To take the following sample test questions as they would be timed in the actual examination, you should allow a
maximum of one hour for each block, for a total of three hours. Please be aware that most examinees perceive the time
pressure to be greater during an actual examination.
A
n answer form for recording answers for this practice is provided on page 63. An answer key is provided on page 64. In
the actual examination, answers will be selected on the screen; no answer form will be provided.
INTRODUCTION TO USMLE STEP 2 CK SAMPLE TEST QUESTIONS
Reference Range SI Reference Intervals
SERUM
General Chemistry:
Electrolytes
Sodium (Na
+
) 136146 mEq/L 136146 mmol/L
Potassium (K
+
) 3.55.0 mEq/L 3.55.0 mmol/L
Chloride (Cl
) 95105 mEq/L 95105 mmol/L
Bicarbonate (HCO
3
) 2228 mEq/L 2228 mmol/L
Urea nitrogen 718 mg/dL 2.56.4 mmol/L
Creatinine 0.61.2 mg/dL 53106 μmol/L
Glucose Fasting: 70100 mg/dL 3.85.6 mmol/L
Random, non-fasting: <140 mg/dL <7.77 mmol/L
Calcium 8.410.2 mg/dL 2.12.6 mmol/L
Magnesium (Mg
2+
) 1.52.0 mg/dL 0.751.0 mmol/L
Phosphorus (inorganic) 3.04.5 mg/dL 1.01.5 mmol/L
Hepatic:
Alanine aminotransferase (ALT) 1040 U/L 1040 U/L
Aspartate aminotransferase (AST) 1238 U/L 1238 U/L
Alkaline phosphatase 25100 U/L 25100 U/L
Amylase 25125 U/L 25125 U/L
Bilirubin, total // direct 0.11.0 mg/dL // 0.00.3 mg/dL 2–17 μmol/L // 0–5 μmol/L
Proteins, total 6.07.8 g/dL 6078 g/L
Albumin 3.55.5 g/dL 3555 g/L
Globulin 2.33.5 g/dL 2335 g/L
Lipids:
Cholesterol
Total Normal: <200 mg/dL <5.2 mmol/L
High: >240 mg/dL >6.2 mmol/L
HDL 4060 mg/dL 1.01.6 mmol/L
LDL <160 mg/dL <4.2 mmol/L
Triglycerides Normal: <150 mg/dL <1.70 mmol/L
Borderline: 151199 mg/dL 1.712.25 mmol/L
Iron Studies:
Ferritin Male: 20250 ng/mL 20–250 μg/L
Female: 10120 ng/mL 10–120 μg/L
Iron Male: 65175 µg/dL 11.6–31.3 μmol/L
Female: 50–170 μg/dL 9.0–30.4 μmol/L
Total iron-binding capacity 250400 µg/dL 44.8–71.6 μmol/L
Transferrin 200360 mg/dL 2.03.6 g/L
USMLE LABORATORY VALUES
Continued on Next Page
10
Reference Range SI Reference Intervals
Endocrine:
Follicle-stimulating hormone Male: 425 mIU/mL 425 IU/L
Female: premenopause 430 mIU/mL 430 IU/L
midcycle peak 1090 mIU/mL 1090 IU/L
postmenopause 40250 mIU/mL 40250 IU/L
Luteinizing hormone Male: 623 mIU/mL 623 IU/L
Female: follicular phase 530 mIU/mL 530 IU/L
midcycle 75150 mIU/mL 75150 IU/L
postmenopause 30200 mIU/mL 30200 IU/L
Growth hormone - arginine stimulation Fasting: <5 ng/mL <5 μg/L
Provocative stimuli: >7 ng/mL >7 μg/L
Prolactin (hPRL) Male: <17 ng/mL <17 μg/L
Female: <25 ng/mL <25 μg/L
Cortisol 0800 h: 5–23 μg/dL 138635 nmol/L
1600 h: 3–15 μg/dL 82413 nmol/L
2000 h: <50% of 0800 h Fraction of 0800 h: <0.50
TSH 0.4–4.0 μU/mL 0.44.0 mIU/L
Triiodothyronine (T
3
) (RIA) 100200 ng/dL 1.53.1 nmol/L
Triiodothyronine (T
3
) resin uptake 25%35% 0.250.35
Thyroxine (T
4
) 5–12 μg/dL 64155 nmol/L
Free T
4
0.91.7 ng/dL 12.021.9 pmol/L
Thyroidal iodine (
123
I) uptake 8%30% of administered dose/24 h 0.080.30/24 h
Intact PTH 1060 pg/mL 1060 ng/L
17-Hydroxycorticosteroids Male: 3.010.0 mg/24 h 8.2–27.6 μmol/24 h
Female: 2.08.0 mg/24 h 5.5–22.0 μmol/24 h
17-Ketosteroids, total Male: 820 mg/24 h 28–70 μmol/24 h
Female: 615 mg/24 h 21–52 μmol/24 h
Immunoglobulins:
IgA 76390 mg/dL 0.763.90 g/L
IgE 0380 IU/mL 0380 kIU/L
IgG 6501500 mg/dL 6.515.0 g/L
IgM 50300 mg/dL 0.53.0 g/L
Other, serum:
Creatinine clearance Male: 97137 mL/min 97137 mL/min
Female: 88128 mL/min 88128 mL/min
Creatine kinase Male: 2590 U/L 2590 U/L
Female: 1070 U/L 1070 U/L
Lactate dehydrogenase 45200 U/L 45200 U/L
Osmolality 275295 mOsmol/kg H
2
O 275295 mOsmol/kg H
2
O
Uric acid 3.08.2 mg/dL 0.180.48 mmol/L
GASES, ARTERIAL BLOOD (ROOM AIR)
PO
2
75105 mm Hg 10.014.0 kPa
PCO
2
3345 mm Hg 4.45.9 kPa
pH 7.357.45 [H
+
] 3644 nmol/L
CEREBROSPINAL FLUID
Cell count 05/mm
3
05 × 10
6
/L
Chloride 118132 mEq/L 118132 mmol/L
Gamma globulin 3%12% total proteins 0.030.12
Glucose 4070 mg/dL 2.23.9 mmol/L
Pressure 70180 mm H
2
O 70180 mm H
2
O
Proteins, total <40 mg/dL <0.40 g/L
USMLE LABORATORY VALUES (continued)
Continued on Next Page
11
Reference Range SI Reference Intervals
HEMATOLOGIC
Complete Blood Count:
Hematocrit Male: 41%53% 0.410.53
Female: 36%46% 0.360.46
Hemoglobin, blood Male: 13.517.5 g/dL 135175 g/L
Female: 12.016.0 g/dL 120160 g/L
Mean corpuscular hemoglobin (MCH) 2535 pg/cell 0.390.54 fmol/cell
Mean corpuscular hemoglobin conc. (MCHC) 31%36% Hb/cell 4.85.6 mmol Hb/L
Mean corpuscular volume (MCV) 80–100 μm
3
80100 fL
Volume
Plasma Male: 2543 mL/kg 0.0250.043 L/kg
Female: 2845 mL/kg 0.0280.045 L/kg
Red cell Male: 2036 mL/kg 0.0200.036 L/kg
Female: 1931 mL/kg 0.0190.031 L/kg
Leukocyte count (WBC) 450011,000/mm
3
4.511.0 × 10
9
/L
Neutrophils, segmented 54%62% 0.540.62
Neutrophils, bands 3%5% 0.030.05
Lymphocytes 25%33% 0.250.33
Monocytes 3%7% 0.030.07
Eosinophils 1%3% 0.010.03
Basophils 0%0.75% 0.000.0075
Platelet count 150,000400,000/mm
3
150400 × 10
9
/L
Coagulation:
Partial thromboplastin time (PTT) (activated) 2540 seconds 2540 seconds
Prothrombin time (PT) 1115 seconds 1115 seconds
D-Dimer ≤250 ng/mL ≤1.4 nmol/L
Other, Hematologic:
Reticulocyte count 0.5%1.5% 0.0050.015
Erythrocyte count (RBC) Male: 4.35.9 million/mm
3
4.35.9 × 10
12
/L
Female: 3.55.5 million/mm
3
3.55.5 × 10
12
/L
Erythrocyte sedimentation rate (Westergren) Male: 015 mm/h 015 mm/h
Female: 020 mm/h 020 mm/h
CD4+ T-lymphocyte count 500/mm
3
0.5 × 10
9
/L
Troponin I ≤0.04 ng/mL ≤0.04 µg/L
Endocrine:
Hemoglobin A
1c
≤6% ≤42 mmol/mol
URINE
Calcium 100300 mg/24 h 2.57.5 mmol/24 h
Osmolality 501200 mOsmol/kg H
2
O 501200 mOsmol/kg H
2
O
Oxalate 8–40 μg/mL 90–445 μmol/L
Proteins, total <150 mg/24 h <0.15 g/24 h
BODY MASS INDEX (BMI) Adult: 1925 kg/m
2
USMLE LABORATORY VALUES (continued)
12
13
B
LOCK 1, ITEMS 140
1. A
21-year-old man comes to student health services because of a 6-month history of increasingly frequent episodes of moderate
chest pain. The first episode occurred while he was sitting in traffic and feeling stressed because he was late for a college class.
At that time, he had the sudden onset of moderate chest pain, a rapid heartbeat, sweating, and nausea. He says he felt as thoug
h
h
e were going to die. The episode lasted approximately 10 minutes. He had a similar episode 1 month later while on a date; the
symptoms were so severe that he abruptly ended the date. During the past 3 weeks, he has experienced two to three episodes
weekly. He says he fears having an episode while in public or on a date, so he has decreased his participation in social activities
and the amount of time he spends outside of his apartment. He has no history of serious illness and takes no medications. He does
not drink alcohol or use other substances. Vital signs are within normal limits. Physical examination discloses no abnormalities.
On mental status examination, he has an anxious mood and full range of affect. Which of the following is the most likely
diagnosis?
(
A) Agoraphobia
(B) Generalized anxiety disorder
(C) Illness anxiety disorder (hypochondriasis)
(D) Social anxiety disorder (social phobia)
(E) Somatic symptom disorder
2. A
35-year-old man is brought to the emergency department by a friend 30 minutes after the sudden onset of right-sided weakness
and difficulty speaking. The symptoms began while he was lifting weights at the gym. He has not had headache or changes in
vision. He has no history of serious illness and takes no medications. He does not smoke cigarettes or drink alcohol. He exercises
regularly. His speech is incoherent, but he can understand what others are saying. Vital signs are within normal limits. No bruits
are heard over the carotid arteries. Cardiopulmonary examination shows no abnormalities. There is swelling and mild tenderness
of the right lower extremity. No cords are palpated. Neurologic examination shows a right facial droop and right upper extremity
weakness. An MRI of the brain shows an acute stroke in the left middle cerebral artery territory. Carotid ultrasonography shows
no abnormalities. Which of the following is the most appropriate next step in diagnosis?
(
A) Adenosine stress test
(B) Cardiac catheterization
(C) Cardiac MRI with gadolinium
(D) CT angiography
(E) Echocardiography with bubble study
3. A
65-year-old man comes to the office because of a 2-year history of progressive shortness of breath on exertion and a 6-mon
th
hi
story of nonproductive cough. He now has shortness of breath when walking to his mailbox. He has not had fever, weight loss,
or chest pain. He has not had recent sick contacts, has had no occupational exposures, and does not own any pets. He has difficult-
to-control atrial fibrillation, hypertension, chronic obstructive pulmonary disease, and migraines. Medications are amiodarone,
warfarin, lisinopril, tiotropium, and propranolol. Temperature is 37.0°C (98.6°F), pulse is 80/min and irregular, respirations ar
e
16/
min, and blood pressure is 110/70 mm Hg. There is no jugular venous distention. Auscultation of the lungs discloses fine
crackles bilaterally, both anteriorly and posteriorly, but no egophony. The remainder of the physical examination discloses n
o
a
bnormalities. An adverse effect of which of the following medications is the most likely cause of these findings?
(A)
Amiodarone
(B) Lisinopril
(C
)
Propranolol
(D) Tiotropium
(E) Warfarin
USMLE STEP 2 CK SAMPLE TEST QUESTIONS
14
4. An 18-month-old boy is brought to the emergency department by his mother 30 minutes after he fell from his bed onto the floor.
Two months ago, he sustained a fracture of the right humerus when he fell while playing in the park. The fracture healed quickly
with immobilization and casting. His mother sustained several bone fractures in early childhood, but she currently does not have
any medical concerns. On arrival, the boy is crying. Pulse is 162/min, respirations are 48/min, and blood pressure is 122/80 mm
Hg. Examination discloses bluish sclera and ecchymoses and tenderness over the right tibia. X-ray of the right lower extremity
confirms a fracture of the tibia. The right lower extremity is immobilized with a cast. Limitation of physical activity is
recommended to prevent future fractures. Given this patient's condition, it is most appropriate for which of the following types
of screening to be done regularly?
(A) Audiography
(B) DEXA scan
(C) Echocardiography
(D) Retinal examination
(E) Serum calcium and vitamin D concentrations
5. A 70-year-old woman is admitted to the hospital because of a 1-hour history of shortness of breath. She has lung cancer and
dementia, Alzheimer type, and has had a decline in mental status during the past month. She lives with her boyfriend of 20 years;
he has cared for her and has taken care of the home since she was diagnosed with dementia 2 years ago. She has one adult son
who lives out of state, but she speaks to him on the phone daily. The patient's neighbor, who is a nurse and a long-term friend,
takes her to all medical appointments and ensures she takes her medications appropriately. The patient is unable to understand
the poor prognosis of her condition. She has not designated a health care power of attorney. Which of the following is the most
appropriate person to make medical decisions for this patient?
(A) Boyfriend
(B) Neighbor
(C) Patient
(D) Physician
(E) Son
6. A 4-month-old female infant is recovering in the hospital 5 days after uncomplicated surgical repair of a ventricular septal defect
via median sternotomy utilizing cardiopulmonary bypass. She was extubated on postoperative day 1 and by postoperative day 4
she was no longer receiving any inotropic infusions or intravenous drugs. She is breast-feeding well; plans are made for discharge
from the hospital the next morning. However, on the next morning she develops a temperature of 38.8°C (101.8°F), cool and
clammy skin, cold distal extremities, delayed capillary refill, and minimal urine output. Cardiac rhythm strip and tracing of arterial
blood pressure are shown. Which of the following is the most appropriate next step in evaluation?
(A) Cardiac catheterization
(B) CT angiography
(C) Echocardiography
(D) Electrophysiology study
(E) MRI of the heart
15
7. Patient Information
Age: 6 years
Sex Assigned at Birth: F
Race/Ethnicity: unspecified
Site of Care: emergency department
History
Reason for Visit/Chief Concern: "My daughter's eyes and legs are swollen."
History of Present Illness:
• 2-week history of cough, congestion, and runny nose that seems to be improving
• parents report the patient's eyes seemed swollen 4 days ago, which they initially attributed to the patient's cold
• 2-day history of swelling over the tops of her feet
• this morning she was unable to put on her shoes because her feet were too swollen
Past Medical History:
• unremarkable
Medications:
• none
Vaccinations:
• up-to-date
Allergies:
• no known drug allergies
Family History:
• mother age 30 years, father age 32 years, and twin sisters age 10 years: alive with no chronic conditions
Physical Examination
Temp
Pulse
Resp
BP
O
2
Sat
Ht
Wt
BMI
37.0°C
140/min
20/min
120/70 mm Hg
99%
115 cm (3 ft 9
in)
25 kg (55 lb)
19 kg/m
2
(98.6°F)
on RA
50th %ile
90th %ile
95th %ile
• Appearance: mildly uncomfortable
• Skin: 2+ pitting edema of both extremities up to the knees
• HEENT: periorbital edema, no erythema; PERRLA; ocular movements are intact
• Pulmonary: clear to auscultation
• Cardiac: regular rhythm; no murmurs
• Abdominal: normoactive bowel sounds; soft, mildly distended, mildly tender to palpation in all quadrants
Diagnostic Studies
Urine
Specific gravity
1.020 (N=1.0031.029)
Protein
Large
Ketones
Negative
Blood
Negative
Leukocyte esterase
Negative
WBCs
Negative
RBCs
5–10/hpf
Question: Which of the following additional laboratory findings are most likely to be decreased in this patient?
(A) Hemoglobin
(B) Serum albumin concentration
(C) Serum C3 and C4 concentrations
(D) Serum triglycerides concentration
(E) Serum urea nitrogen concentration
16
8. A 9-year-old girl is brought to the clinic for a routine examination. She has a 4-year history of asthma. Her only medication is
inhaled albuterol with a spacer as needed. She uses it only occasionally and has not used it at all during the past 4 weeks. Her
father reports that his daughter has an episode of coughing that awakens her at night once weekly. He also says that his daughter
has a sedentary lifestyle because "she coughs if she runs too much." No one in the family smokes cigarettes, and there are no
pets. Gas heaters are used in the home. The patient is at the 25th percentile for height and 90th percentile for weight and BMI.
Respirations are 16/min. Lungs are clear to auscultation. There is no clubbing of the digits. The remainder of the examination
shows no abnormalities. Which of the following is the most appropriate next step in management?
(A) Add oral theophylline to the regimen
(B) Add fluticasone by metered-dose inhaler with a spacer to the regimen
(C) Add salmeterol by metered-dose inhaler with a spacer to the regimen
(D) Begin a 5-day course of oral prednisone
(E) No change in management is necessary
9. A 30-year-old man comes to the office to establish primary care. Medical history is unremarkable and he takes no medications.
His father had a myocardial infarction at age 48 years. The patient has smoked one-half pack of cigarettes daily for 10 years. He
does not drink alcoholic beverages. He is 168 cm (5 ft 6 in) tall and weighs 82 kg (180 lb); BMI is 29 kg/m
2
. Vital signs are
within normal limits. The patient is not in distress. Physical examination discloses no abnormalities. Smoking cessation is
recommended. Which of the following is the most appropriate screening study for this patient at this time?
(A) ECG
(B) Fasting serum lipid studies
(C) Serum chemistry profile
(D) No screening studies are indicated
10. An 82-year-old woman comes to the office because of a 1-month history of increasing numbness of her feet. She has no history
of serious illness. She has taken over-the-counter calcium carbonate tablets for intermittent abdominal pain during the past 40
years; she takes no other medications. She has drunk two glasses of wine with dinner nightly for 50 years. On examination,
gastrocnemius deep tendon reflexes are absent. Babinski sign is absent bilaterally. Light touch to the distal lower extremities and
feet produces a tingling sensation. Sensation to vibration over the great toes is severely decreased. There is a mild to moderate
decrease in proprioception of the great toes. Which of the following is most likely to prevent progression of these neurologic
findings?
(A) Alcohol cessation
(B) Calcium carbonate cessation
(C) Folic acid supplementation
(D) Niacin supplementation
(E) Vitamin B
1
(thiamine) supplementation
(F) Vitamin B
12
(cyanocobalamin) supplementation
17
11. A 50-year-old man comes to the office for a health maintenance examination. He says he has felt well and reports no symptoms.
Medical history is unremarkable. He takes no medications. He uses up to three cans of chewing tobacco weekly and occasionally
sleeps with tobacco in his mouth. He says he would like to quit but cannot seem to do it on his own. Vital signs are within normal
limits. Examination shows a whitish discoloration that measures approximately 1 cm in diameter on the buccal mucosa. The
tongue has a brownish discoloration. Palpation of the neck discloses no lymphadenopathy. In addition to encouraging the patient
to quit using chewing tobacco, which of the following is the most appropriate next step in management?
(A) Prescribing oral nystatin suspension
(B) Surgical biopsy of the oral lesion
(C) Swabbing of the white area and sending for cytology
(D) Observation only
12. A 48-year-old man is brought to the emergency department by ambulance 45 minutes after he collapsed at home. He did not have
loss of consciousness. He has a 6-week history of shortness of breath and severe fatigue when he walks 20 to 30 feet. During the
past 6 weeks, he also has had a 5-kg (11-lb) weight gain despite a decreased appetite. He has no history of serious illness. He
cannot remember his last visit to a physician. His only medication is ibuprofen for intermittent mild headaches. His father died
of “heart problems” at the age of 78 years. The patient has smoked one pack of cigarettes daily for 22 years. He has consumed
24 cans of beer weekly for 20 years. He works as a painting and drywall contractor but was unable to work last week because of
fatigue. Temperature is 36.8°C (98.2°F), pulse is 100/min and regular, respirations are 16/min, and blood pressure is 90/55 mm
Hg while supine. Pulse oximetry on 4 L/min of oxygen by nasal cannula shows an oxygen saturation of 91%. On examination,
crackles are heard halfway up the lung fields bilaterally. An S
3
is heard; no murmurs are heard. The liver span is 5 cm. There is
moderate pitting edema of the calves and ankle swelling bilaterally. Echocardiography is most likely to show which of the
following findings?
(A) A large pericardial effusion
(B) Dilated cardiomyopathy
(C) Left ventricular hypertrophy
(D) Paradoxical septal motion
(E) Regional wall motion abnormality
13. A 68-year-old man comes to the clinic because of a 6-month history of frequent falls. He sustained a fracture of the left wrist 4
weeks ago during a fall. He has not had loss of consciousness before or during the falls. He says that turning and pivoting often
cause him to fall, and he has a tendency to fall backward when walking. He has Parkinson disease treated with carbidopa-levodopa
and entacapone. Vital signs are within normal limits. Examination shows masked facies and a resting tremor of the right upper
extremity. Romberg sign is absent. When the patient stands still and is pulled backward, he is unable to maintain his posture.
Which of the following is the most appropriate intervention to decrease this patient's risk for future falls?
(A) Biofeedback
(B) Physical therapy
(C) Pramipexole therapy
(D) Ropinirole therapy
(E) Rotigotine therapy
18
14. A 6-year-old boy is brought to the office by his parents as a new patient. Three months ago, he and his 3-year-old sister and 4-
year-old brother were adopted from a Russian orphanage, where they had lived for 2 years. Their biological parents died in a
motor vehicle collision; they have no other relatives. The adoptive parents say they were told that the children had stable
caregivers, age-appropriate stimulation, and schooling at the orphanage. The children were shy initially on meeting them but now
seem comfortable. They play well with each other; the patient is protective of his siblings. His medical and developmental history
prior to 2 years ago is unknown. He has been learning English slowly; his parents hope he will learn enough by the end of the
summer to begin kindergarten, a grade below that of his peers. Although he can perform basic mathematics, he is unable to read,
even in Russian. Results of receptive language testing are consistent with an average IQ. His daily living skills are appropriate
for age. He is at the 10th percentile for height and weight. During the examination, the patient’s adoptive mother, who is fluent
in Russian, translates for him. Physical examination shows no abnormalities. On mental status examination, he is shy and
maintains intermittent eye contact. He slowly answers questions with brief statements. He smiles and says he is excited and
pleased about being adopted. Results of laboratory studies are within the reference ranges. Which of the following is the most
likely diagnosis?
(A) Autism spectrum disorder
(B) Fetal alcohol syndrome
(C) Intellectual developmental disorder
(D) Learning disorder
(E) Post-traumatic stress disorder
(F) Reactive attachment disorder
15. A 17-year-old boy is brought to the emergency department because of a 1-hour history of progressive difficulty breathing and a
2-hour history of an itchy, red rash over his trunk, arms, and legs. He has not used new soaps, detergents, or lotions during the
past week. Thirty-six hours ago, he began a 1-week course of amoxicillin for an ear infection; he has taken four doses since that
time. He has no other history of serious illness and no known allergies. He takes no other medications. Pulse is 100/min,
respirations are 28/min, and blood pressure is 90/60 mm Hg. Pulse oximetry on room air shows an oxygen saturation of 95%. He
appears anxious and is in moderate respiratory distress. Examination shows an erythematous, raised, demarcated rash over the
trunk and all extremities. Which of the following is the most appropriate next step in management?
(A) Administration of albuterol
(B) Administration of diphenhydramine
(C) Administration of epinephrine
(D) Complete blood count
(E) Observation only
16. Two weeks after undergoing open cholecystectomy for gangrenous cholecystitis, a 47-year-old woman comes to the clinic for
removal of surgical staples. She was treated with oral vancomycin for Clostridioides difficile colitis during hospitalization. She
has type 2 diabetes mellitus. Current medications are metformin and pantoprazole. She reports loose stools four times daily during
the past 2 weeks. In addition to wearing a clean isolation gown, which of the following is the most appropriate precaution for the
physician to take?
(A) Washing hands thoroughly with alcohol gel
(B) Washing hands thoroughly with soap and water
(C) Wearing a surgical mask
(D) Wearing sterile gloves and preparing the staple skin line with chlorhexidine
(E) No additional precautions are necessary
19
Question
In patients with opioid use disorder, what is the relative efficacy of buprenorphine and methadone maintenance therapy?
Review scope
Included studies compared buprenorphine maintenance therapy (BMT), >1 mg/day, with methadone maintenance therapy (MMT),
≥20 mg/day, or placebo (including 1 mg/day of buprenorphine) in patients who were dependent on heroin or other opioids. Studies of
pregnant women and those assessing buprenorphine or methadone for detoxification but with no maintenance phase were excluded.
Outcomes included treatment retention and urinalysis-confirmed use of morphine, cocaine, and benzodiazepines.
Review methods
MEDLINE and EMBASE/Excerpta Medica (2003 to Jan 2013), Cochrane Central and Cochrane Library (2013, Issue 1), PsycLIT,
Current Contents, several other databases, ClinicalTrials.gov and other electronic sources of ongoing trials, conference proceedings,
Library of Congress databases, national focal points for drug research, and reference lists were searched for randomized controlled
trials (RCTs). Authors were consulted. 31 RCTs (n=5430), ranging in size from 40 to 736 patients, met the selection criteria. Duration
of study interventions ranged from 2 weeks to 52 weeks. 20 trials compared BMT with MMT, and 11 compared BMT with placebo.
Main results
As shown in the table below, low-, medium-, and high-dose BMT increased treatment retention more than placebo; BMT did not
differ from equivalent doses of MMT for retention or had lower retention.
High-dose (standard mean difference −1.17, 95% CI −1.85 to −0.49), but not medium- or low-dose, BMT decreased morphine use
more than placebo. Medium- and flexible-dose BMT and MMT did not differ for morphine use or cocaine use. BMT vs MMT and
high-dose BMT vs placebo did not differ for benzodiazepine use.
Buprenorphine maintenance therapy (BMT) vs methadone maintenance therapy (MMT) or placebo for treatment retention in
patients with opioid use disorder
Number of
Weighted
Retention
Comparisons*
Trials (n)
Rates
RBI (95% CI)
NNT (95% CI)
High-dose BMT vs placebo
5 (1001)
72% vs 40%
82% (15 to 190)
4 (3 to 7)
Medium-dose BMT vs placebo
4 (887)
66% vs 38%
74% (6 to 187)
4 (2 to 34)
Low-dose BMT vs placebo
5 (1131)
60% vs 40%
50% (19 to 88)
5 (4 to 10)
RBR (95% CI)
NNH (95% CI)
High-dose BMT vs high-dose MMT
1 (134)
5.2% vs 6.6%
21% (−216 to 20)
Not significant
Medium-dose BMT vs medium-dose MMT
7 (780)
43% vs 48%
13% (−10 to 31)
Not significant
Flexible-dose BMT vs flexible-dose MMT
11 (1391)
50% vs 63%
17% (5 to 27)
8 (5 to 25)
Low-dose BMT vs low-dose MMT
3 (253)
38% vs 56%
33% (13 to 48)
6 (4 to 17)
CI = confidence interval; NNH = number needed to harm; NNT = number needed to treat; RBI = relative benefit increase;
RBR = relative benefit reduction.
*Flexible dose = dose is titrated within a broad dose range according to patient preference; low-dose BMT = 2 to 6 mg;
medium-dose BMT = 7 to 15 mg; high-dose BMT = 16 mg; medium-dose MMT = 40 to 85 mg.
Conclusions
In patients with opioid use disorder, buprenorphine is more efficacious than placebo, but less efficacious than methadone therapy, for
treatment retention.
Source of funding: Australian Government Department of Health.
Structured abstract based on: Mattick RP, Breen, C, Kimber J, Davoli M. Buprenorphine maintenance versus placebo or
methadone maintenance for opioid dependence. Cochrane Database Syst Rev. 2014;(2):CD002207. 24500948
The abstract displayed on this page is for use with items #17–19 on the following page.
20
17. A 38-year-old woman comes to the clinic for follow-up examination. She has opioid use disorder and currently takes methadone
40 mg daily as maintenance therapy. She says the methadone is causing mild sedation that is interfering with her job. She would
like to discontinue methadone maintenance therapy but is concerned about heroin relapse. She says she also takes a friend’s
valium when she is feeling “stressed out.” The physician discusses alternative maintenance therapy strategies with the patient.
The physician discusses the pros and cons of medium- or high-dose buprenorphine maintenance therapy compared with medium-
or high-dose methadone therapy with the patient. Based on this abstract, which of the following is the most appropriate
information for the physician to convey to the patient?
(A) Buprenorphine is 13% and 21% less effective than methadone at medium and high doses, respectively
(B) Buprenorphine is a better maintenance strategy if the patient is also regularly using benzodiazepines
(C) Methadone is statistically more likely to result in treatment retention
(D) Neither drug is more effective than placebo
(E) There is an unclear difference in efficacy between the two drugs at these doses
18. Which of the following aspects of the study is most likely to limit confidence in the authors’ conclusions?
(A) Exclusion of patients using methadone for acute detoxification
(B) Exclusion of pregnant patients
(C) Inclusion of randomized controlled trials only
(D) Inclusion of trials that compared active drug with placebo
(E) Inclusion of trials with an intervention duration of 2 weeks
(F) Use of urinalysis to determine treatment retention
19. Which of the following is most likely to bias the results of this study?
(A) Earlier diagnosis of relapse in patients receiving methadone maintenance therapy
(B) Exclusion criteria of studies used for the review
(C) Likelihood that more trials with positive results will be published
(D) Source of funding
(E) Variety of sample sizes of the included trials
END OF SET
20. A 57-year-old man is brought to the emergency department by ambulance 30 minutes after he had loss of consciousness. His wife
says he was making dinner when he fell on the kitchen floor. He has hypertension and his only medication is lisinopril. He has
no history of operative procedures. En route, his temperature was 37.2°C (99.0°F), pulse was 90/min, respirations were 10/min
and labored, and blood pressure was 180/100 mm Hg. Pulse oximetry on 40% oxygen by face mask showed an oxygen saturation
of 99%. On arrival, he is unresponsive. His Glasgow Coma Scale score is 3. He is intubated and mechanically ventilated.
Temperature is 37.2°C (99.0°F), pulse is 90/min, ventilatory rate is 10/min, and blood pressure is 180/100 mm Hg. Pulse oximetry
on an F
IO
2
of 0.4 shows an oxygen saturation of 98%. On examination, Doll's eye (oculocephalic) maneuver shows absent eye
movements, and the corneal reflex is absent. On ice-water caloric testing, there is no nystagmus or deviation of the eyes toward
the ear being irrigated. CT scan of the head shows a ruptured cerebral aneurysm. When the patient's wife is notified of the patient's
condition, she says he wanted to be an organ donor but does not have an advance directive. Which of the following is the most
appropriate next step in management?
(A) Consult with the hospital ethics committee
(B) Contact the organ bank for potential donation
(C) Determine if there are any patients awaiting organ donation in the hospital
(D) Explain that the patient is not a candidate for organ donation because he does not have an advance directive
(E) Explain that the patient is not a candidate for organ donation because of his history of hypertension
Items #17–19 refer to the abstract displayed on the previous page.
21
21. A 36-year-old woman, gravida 2, para 1, at 39 weeks’ gestation is admitted to the hospital in labor. She reports painful contractions
that occur every 2 to 3 minutes and last 60 seconds. She has not had vaginal bleeding. She has no history of serious illness, and
pregnancy has been uncomplicated. Temperature is 37.0°C (98.6°F), pulse is 84/min and regular, and blood pressure is 100/70
mm Hg. The abdomen is nontender and consistent in size with a 39-week gestation. External fetal monitoring shows a heart rate
of 150/min with moderate variability, several spontaneous accelerations, and no decelerations. An external tocometer shows
regular uterine contractions every 2 to 3 minutes. The cervix is 6 cm dilated and 100% effaced; the vertex is at 0 station.
Ultrasonography shows the fetus in a cephalic presentation. The membranes are artificially ruptured, yielding copious clear fluid.
Thirty minutes later, the cervix is 9 cm dilated and 100% effaced; the vertex is at +1 station. A fetal heart tracing is shown. Which
of the following is the most appropriate next step in management?
(A) Advising the patient to begin pushing
(B) Amnioinfusion
(C) Expectant management
(D) Forceps-assisted vaginal delivery
(E) Immediate cesarean delivery
22. A 27-year-old woman comes to the office because of a 3-week history of nasal congestion, cough productive of sputum,
intermittent moderate headache, and intermittent moderate pain over her cheeks. She has had similar symptoms three to four
times yearly during the past 5 years. The symptoms last 2 to 8 weeks and generally resolve spontaneously. She has had two
episodes of bacterial pneumonia during the past 5 years; both episodes resolved with antibiotic therapy. Her only current
medication is acetaminophen as needed for headache. Vital signs are within normal limits. Examination shows no abnormalities.
Serum study results show decreased IgA, IgG, and IgM concentrations. Antibiotic therapy is begun, and the patient’s condition
improves. Which of the following is most likely to decrease the likelihood of recurrent infection in this patient?
(A) Daily inhaled tobramycin therapy
(B) Daily intranasal glucocorticoid therapy
(C) Daily trimethoprim-sulfamethoxazole therapy
(D) Intranasal influenza virus vaccine administration
(E) Monthly immune globulin replacement therapy
22
23. A 37-year-old man comes to the office because of a 2-year history of mild to moderate fatigue. The fatigue waxes and wanes
without relation to activity. He has no history of serious illness and takes no medications. He drinks three glasses of whiskey
weekly. He has had two lifetime female sexual partners; he uses condoms consistently. He immigrated to the United States from
China 15 years ago. Vital signs are within normal limits. Examination discloses no abnormalities. Results of laboratory studies
are shown:
Serum
Creatinine
1.2 mg/dL
ALT
85 U/L
AST
60 U/L
HBsAg
positive
Anti-HCV
negative
Anti-HBs
negative
IgG anti-HBc
positive
Blood
Hematocrit
35%
Hemoglobin
12 g/dL
MCV
82 µm
3
Without treatment of his current condition, this patient is most likely to develop which of the following?
(A) Amyloidosis
(B) Essential mixed cryoglobulinemia
(C) Hepatocellular carcinoma
(D) Membranoproliferative glomerulonephritis
(E) Polyarteritis nodosa
(F) Sjögren syndrome
24. A 10-year-old boy is brought to the office because of moderate right knee pain. He is in no distress. Medical history is remarkable
for hemophilia A. Examination of the right knee shows warmth and swelling; there is pain with flexion. Range of motion is
limited by pain and swelling. Which of the following is the most appropriate therapy for this patient?
(A) ADH (vasopressin)
(B) Factor VIII concentrate
(C) Factor IX concentrate
(D) Ferrous sulfate
(E) Fresh frozen plasma
25. A 72-year-old man comes to the clinic because of a 3-week history of visual changes in his right eye. During this period, he has
noticed that the lines of his daily crossword puzzle look curved, and the blinds in his apartment appear wavy. He wears magnifying
lenses for reading. He has not had pain in his eye or photophobia. He has no history of trauma to the area. On examination, visual
acuity is 20/200 in the right eye and 20/100 in the left eye. Both pupils are round and reactive to light. Palpation of both globes
through closed eyelids shows no abnormalities. There is normal red reflex bilaterally. The lenses appear clear. Which of the
following is the most likely diagnosis?
(A) Cataracts
(B) Central retinal artery occlusion
(C) Closed-angle glaucoma
(D) Macular degeneration
(E) Temporal arteritis
23
26. A 28-year-old man comes to the clinic because of a 2-day history of a red, severely painful lesion on his left forearm. He says that
he felt stinging pain as he put his shirt on in the dark 2 days ago; the pain was so severe he immediately threw down the shirt and
saw a spider running across the floor. He returned yesterday from a 1-week trip to the mountains in the southeastern United
States. He has type 1 diabetes mellitus treated with lisinopril and insulin. He does not drink alcoholic beverages. He is sexually
active with two male partners and uses condoms inconsistently. Vital signs are within normal limits. A photograph of the lesion
is shown. No other abnormalities are noted. Which of the following physical findings is most likely to appear during the next 24
hours?
(A) Areas of eschar at the center of the wound site
(B) Gangrene of the distal aspect of the left extremity
(C) Generalized upper extremity edema
(D) Palpable purpuric lesions across the trunk and upper extremities
(E) Rapidly spreading erythema around the wound site
(F) Tender, erythematous streaks up to the epitrochlear nodes
27. A 37-year-old man comes to the clinic for a health maintenance examination prior to employment. He feels well. He has no history
of serious illness and takes no medications. One year ago, his blood pressure was 136/85 mm Hg. He is 173 cm (5 ft 8 in) tall
and weighs 100 kg (220 lb); BMI is 33 kg/m
2
. Today, his blood pressure is 138/87 mm Hg. Examination shows no abnormalities.
In addition to recommending weight loss, which of the following is the most appropriate next step to prevent cardiovascular
morbidity in this patient?
(A) ACE inhibitor therapy
(B) Calcium supplementation
(C) DASH diet
(D) Fish oil supplementation
(E) Thiazide diuretic therapy
24
28. A 39-year-old man comes to the office because of a 6-week history of mild fatigue and increased thirst and urination. During this
time, he also has had a 6.8-kg (15-lb) weight loss despite no change in appetite. He is otherwise asymptomatic. He has no history
of serious illness and takes no medications. He is 180 cm (5 ft 11 in) tall and weighs 61 kg (135 lb); BMI is 19 kg/m
2
. Temperature
is 37.0°C (98.6°F), pulse is 80/min, respirations are 18/min, and blood pressure is 118/70 mm Hg. Examination shows no
abnormalities. Results of laboratory studies are shown:
Serum
Na
+
135 mEq/L
K
+
4.2 mEq/L
Cl
100 mEq/L
Urea nitrogen
25 mg/dL
Glucose
578 mg/dL
Blood
Hemoglobin A
1c
10.7%
In addition to recommending dietary modification, which of the following is the most appropriate next step in management?
(A) Glyburide therapy
(B) Insulin therapy
(C) Metformin therapy
(D) Pioglitazone therapy
(E) Sitagliptin therapy
29. Two days after admission to the hospital for treatment of a fractured femur and closed head injury sustained in a motor vehicle
collision, an 8-year-old girl has a brief, generalized tonic-clonic seizure. CT scan of the head on admission disclosed no
abnormalities. She has no history of serious illness. Oral acetaminophen was initiated for pain, and the patient underwent open
reduction and internal fixation of the fracture. She receives no routine medications. Today, she appears drowsy but responds to
questions appropriately. She is at the 55th percentile for height, weight, and BMI. Temperature is 36.8°C (98.2°F), pulse is
92/min, respirations are 24/min, and blood pressure is 106/64 mm Hg. Pulse oximetry on room air shows an oxygen saturation
of 95%. Pupils are equal and reactive to light and accommodation. Ocular movements are full. Muscle strength is 5/5, and deep
tendon reflexes are normal. Results of serum studies are shown:
Na
+
122 mEq/L
K
+
3.8 mEq/L
Cl
94 mEq/L
HCO
3
24 mEq/L
Glucose
80 mg/dL
Calcium
8.8 mg/dL
Which of the following is the most appropriate next step in management?
(A) Administer a bolus of intravenous 3% saline
(B) Administer subcutaneous vasopressin
(C) Initiate fosphenytoin therapy
(D) Initiate therapy with 0.9% saline at 1.5 times the maintenance requirements
(E) Order EEG
(F) Order MRI of the brain
25
30. A 43-year-old man comes to the office because of a 2-week history of fatigue and mild pain in his right armpit. He has not had
fever or weight loss. Medical history is remarkable for type 2 diabetes mellitus and hypercholesterolemia. Medications are
metformin, simvastatin, and aspirin. He is a social worker and frequently visits clients in homeless shelters. He lives alone with
two dogs and a cat and has one male sexual partner. Vital signs are within normal limits. He appears well. Examination of the
right upper extremity shows an erythematous axilla with a 4-cm, mobile, tender, nonfluctuant axillary lymph node and a tender,
1-cm epitrochlear lymph node. Abdominal examination shows no hepatosplenomegaly. Which of the following is the most likely
diagnosis?
(A) Castleman disease
(B) Cat-scratch disease
(C) Hidradenitis suppurativa
(D) T-cell lymphoma
(E) Tuberculosis
31. A 2-year-old girl is brought to the office by her parents for a well-child examination. She was adopted 2 months ago. She has no
history of serious illness and receives no medications. Vital signs are within normal limits. Examination shows no abnormalities.
After the examination, the parents ask when they should tell their daughter about the adoption. Which of the following is the most
appropriate advice regarding the best time to tell this patient?
(A) After she turns 18 years of age
(B) As early as possible, even if she cannot process the whole experience
(C) Once the seal on the adoption records has been lifted
(D) Right before she is enrolled in kindergarten
(E) When she learns that she is not biologically related to her parents
32. A surgical intensive care unit observes that the number of days that patients have central venous catheters in place and the number
of bloodstream infections have been increasing. A multidisciplinary team from the unit would like to decrease the number of
days that central venous catheters are in place in order to improve performance in this area. Which of the following is the most
appropriate next step?
(A) Ask clinical providers in the unit to identify what they believe to be the most common factors leading to nonremoval of
central venous catheters
(B) Change all central venous catheters every 5 days
(C) Place stickers on all intensive care unit doors reminding staff to reassess the need for a central line
(D) Plan a randomized controlled trial to test the effects of a reminder to reassess the need for a central line on a daily goals
sheet implemented during rounds
33. A 17-year-old girl comes to the office because of a 3-month history of daily moderate epigastric pain that is more severe after
meals. Three weeks ago, she had watery diarrhea for 2 days, which resolved spontaneously. She has not had vomiting. She has
no history of serious illness and takes no medications. Vital signs are within normal limits. Examination, including abdominal
examination, shows no abnormalities. Which of the following is most likely to confirm the diagnosis?
(A) CT scan of the abdomen
(B) Endoscopy
(C) Examination of the stool for ova and parasites
(D) Stool culture
(E) Upper gastrointestinal series
26
34. A 23-year-old man comes to the emergency department because of a 1-hour history of intermittent coughing spasms that began
suddenly while he was attempting to clean mold from his garage with a mixture of acetic acid and cleaning agents. The cough is
not productive. He reports shortness of breath only during the coughing spasms. Medical history is unremarkable and he takes no
medications. He does not smoke cigarettes, drink alcoholic beverages, or use other substances. He is 180 cm (5 ft 11 in) tall and
weighs 75 kg (165 lb); BMI is 23 kg/m
2
. Vital signs are within normal limits. Pulse oximetry on room air shows an oxygen
saturation of 94%. The patient has periodic paroxysmal coughing during the examination. Oropharynx is clear. Auscultation of
the lungs discloses bilateral wheezes throughout all lung fields. There is no use of accessory muscles of respiration. The remainder
of the physical examination discloses no abnormalities. Chest x-ray shows mild hyperexpansion of the lungs bilaterally. Results
of arterial blood gas analysis on room air are shown:
PO
2
72 mm Hg
PCO
2
38 mm Hg
pH
7.41
HCO
3
23 mEq/L
Which of the following is the most appropriate initial step in management?
(A) Albuterol therapy
(B) Bronchoscopy
(C) CT scan of the chest
(D) Intubation
(E) Methylprednisolone therapy
35. An 87-year-old man is admitted to the hospital because of a 4-day history of moderate abdominal cramps, nausea, and vomiting.
He has not had a bowel movement or passed gas in the past 24 hours. He has colon cancer metastatic to the lungs, liver, and
peritoneum. Eighteen months ago, he underwent a low anterior resection for T3N2 rectal cancer. Six months ago, exploratory
laparotomy showed diffuse carcinomatosis and bowel obstruction, and an ileotransverse bypass was performed. The patient and
his family were informed of the findings and advised that he would not be a candidate for a laparotomy in the future because
there would be no surgical options to treat another obstruction. The patient has no other history of serious illness. Medications
are 5-fluorouracil and bevacizumab. Temperature is 37.2°C (99.0°F), pulse is 100/min, respirations are 20/min, and blood
pressure is 130/80 mm Hg. The abdomen is distended and mildly tender to palpation. There are no peritoneal signs. CT scan of
the abdomen shows a mid-jejunal small-bowel obstruction, multiple intraperitoneal tumor deposits, and ascites. The physician
recommends insertion of nasogastric and percutaneous endoscopic gastrostomy tubes. The patient and his family request that the
patient undergo a laparotomy and intestinal bypass. Which of the following is the most appropriate next step in management?
(A) Abide by the wishes of the patient and his family
(B) Alter the chemotherapy regimen
(C) Explain the futility of the operation to the patient and his family
(D) Offer to transfer the patient to another facility
(E) Suggest external beam radiation therapy
36. A 62-year-old woman comes to the office because of a 3-month history of anxiety, insomnia, and frequent bowel movements.
She has had a 9-kg (20-lb) weight loss during this period. Pulse is 90/min and irregularly irregular, respirations are 22/min, and
blood pressure is 150/65 mm Hg. She is restless. The thyroid gland is diffusely enlarged. A bruit is heard over the thyroid.
Examination shows palmar erythema; some of her fingernails are separated from the nail beds. There is widening of the palpebral
fissures and thickening of the skin over the dorsum of the feet with a peau d'orange appearance. There is a fine tremor. Which of
the following is the most likely diagnosis?
(A) Addison disease
(B) Carcinoid syndrome
(C) Cushing syndrome
(D) Pheochromocytoma
(E) Thyrotoxicosis
27
37. Patient Information
Age: 28 years
Gender: M, self-identified
Race/Ethnicity: unspecified
Site of Care: clinic
History
Reason for Visit/Chief Concern: follow-up 1 week after undergoing colonoscopy because of a 3-month history of
hematochezia and iron deficiency anemia
History of Present Illness:
• 3-month history of three loose bowel movements daily
• no fever or abdominal pain
• since colonoscopy, no bright red blood per rectum
• reports no new symptoms today
Past Medical History:
• generalized anxiety disorder
Medications:
• citalopram
Allergies:
• no known drug allergies
Family History:
• mother and father alive with no chronic conditions
Psychosocial History:
• has smoked two packs of cigarettes daily for 10 years
• does not drink alcoholic beverages
Physical Examination
Temp
Pulse
Resp
BP
O
2
Sat
Ht
Wt
BMI
36.1°C
76/min
14/min
100/62 mm Hg
170 cm
59 kg
20 kg/m
2
(97.0°F)
(5 ft 7 in)
(130 lb)
• Abdominal: scaphoid; normoactive bowel sounds; liver span is 9 cm by percussion; mild tenderness to palpation over the right
lower quadrant without rebound or guarding; no hepatosplenomegaly
Diagnostic Studies
• colonoscopy: areas of coalesced ulcers with normal appearing mucosa between, involving the ileum
Question: Which of the following is the most appropriate recommendation for this patient considering his condition?
(A) Annual screening for lymphoproliferative disorders
(B) Discontinuation of citalopram therapy
(C) Prophylactic colectomy
(D) Smoking cessation
38. A 19-year-old man is brought to the emergency department (ED) by ambulance 15 minutes after he was stabbed in the chest
during a fight. The stab wound is located medially to the left nipple. At the scene, pulse was 105/min, respirations were 18/min,
and blood pressure was 120/80 mm Hg. Pulse oximetry on room air showed an oxygen saturation of 96%. Two large-bore
intravenous catheters were placed en route to the ED, and lactated Ringer solution was initiated. On arrival in the trauma bay, the
patient develops a pulse of 130/min and blood pressure of 80/40 mm Hg. There is jugular venous distention. Results of focused
assessment with sonography for trauma (FAST) are most likely to lead to which of the following next steps for this patient?
(A) CT scan of the chest and abdomen with contrast
(B) Exploratory laparotomy
(C) Needle decompression and chest tube placement
(D) Pericardiocentesis
(E) Peritoneal lavage
28
39. A 47-year-old woman is scheduled to undergo right knee arthroscopy and partial medial meniscectomy. Medical history is
otherwise unremarkable. At her preoperative examination 2 weeks ago, her surgeon discussed the risks and benefits of the
procedure, and both parties signed the operative consent form. On the patient's arrival at the hospital, the paperwork is reviewed.
She is taken into the operating room and sedated, and the anesthesiology team starts a spinal anesthetic. After the patient's right
lower extremity is prepped for the procedure, a surgical pause is taken to review the patient's name and consent form and to
confirm the site of surgery. The operating room team is unable to locate the consent form for review. No markings are present on
the patient's extremities to indicate the site of the procedure. The surgeon says that he remembers reviewing the consent form
with the patient and seeing her sign it. The patient's husband is in the waiting room. Which of the following is the most appropriate
course of action?
(A) Ask the patient if she recalls signing the consent form and, if so, proceed with the operation
(B) Ask the patient to sign a new consent form and proceed with the operation
(C) Do not proceed, and remove the patient from the operating room
(D) Obtain verbal consent from the patient's husband before proceeding with the operation
(E) Proceed with the operative procedure as planned
40. A 30-year-old man comes to the physician because he and his wife have been unable to conceive during the past 2 years. Previous
evaluation of his wife showed no abnormalities. He has not been exposed to pesticides, heavy metals, radiation, or testicular
overheating. Physical examination shows no abnormalities. Serum studies show a testosterone concentration within the lower
reference range. A diagnosis of oligospermia is made. Which of the following is the most appropriate initial action by the
physician?
(A) Ask the patient about his alcohol intake, smoking, and stress level
(B) Educate the patient about declining sperm counts with increased age
(C) Prescribe a phosphodiesterase inhibitor
(D) Provide a topical testosterone gel
(E) Recommend in vitro fertilization
29
BLOCK 2, ITEMS 41–80
41. A 40-year-old man comes to the office during the summer because of a 3-day history of fever, diffuse muscle aches, fatigue, and
sore throat. He has had no sick contacts. Medical history is unremarkable and he takes no medications. He received the influenza
virus vaccine this year. Ten days ago, he returned from a trip to Malaysia, where he says he enjoyed the local food and nightlife.
Temperature is 38.8°C (101.8°F), pulse is 90/min, respirations are 16/min, and blood pressure is 126/70 mm Hg. The remainder
of the examination shows no abnormalities. Results of laboratory studies are shown:
Blood
Hemoglobin
10 g/dL
MCV
90 μm
3
WBC
2400/mm
3
Neutrophils, segmented
60%
Neutrophils, bands
5%
Lymphocytes
5%
Monocytes
20%
Eosinophils
4%
Basophils
6%
Platelet count
350,000/mm
3
Red cell distribution width
14%
Monospot test result is negative. Which of the following is the most appropriate next step in diagnosis?
(A) Cytomegalovirus serologic testing
(B) Epstein-Barr virus serologic testing
(C) Hemagglutination inhibition assay
(D) HIV RNA polymerase chain reaction testing
(E) Serum hepatitis B surface antigen testing
42. A 22-year-old man is brought to the emergency department by his girlfriend 30 minutes after she found him unconscious in his
home. She last saw him yesterday and he appeared well. He has a history of cocaine and heroin use, and also use of prescription
narcotic analgesics that he obtains from friends. On arrival, he is lethargic. Temperature is 35.0°C (95.0°F), pulse is 68/min,
respirations are 8/min, and blood pressure is 96/50 mm Hg. Pulse oximetry on room air shows an oxygen saturation of 96%. The
left lower extremity is cyanotic and cold to the touch. The remainder of the examination shows no abnormalities. Results of serum
studies are shown:
K
+
5.5 mEq/L
HCO
3
18 mEq/L
Creatinine
1 mg/dL
Creatine kinase
50,000 U/L
This patient is at greatest risk for which of the following conditions?
(A) Acute kidney injury
(B) Acute liver failure
(C) Cardiac arrhythmia
(D) Hypocalcemia
(E) Hypophosphatemia
USMLE STEP 2 CK SAMPLE TEST QUESTIONS
30
43. A 62-year-old woman is brought to the emergency department (ED) by her husband because of a 3-day history of increasing
confusion. She has a 20-year history of bipolar disorder well controlled with medication for the past 5 years. Her husband forgot
to bring her medication bottles and cannot remember which medication she takes. He reports that she has recently had mild
headaches and has taken acetaminophen as needed. On questioning, he says that there has been no particular change in their
routine, but during the past 2 days they have been outside playing tennis more than usual because of the onset of sunny, warm
weather. While in the ED, the patient has a generalized tonic-clonic seizure lasting 2 minutes. She is 163 cm (5 ft 4 in) tall and
weighs 72 kg (160 lb); BMI is 27 kg/m
2
. Pulse is 104/min, respirations are 18/min, and blood pressure is 140/90 mm Hg.
Examination shows dry mucous membranes. ECG shows a second-degree atrioventricular block. Which of the following is most
likely responsible for this patient's confusion?
(A) Acetaminophen
(B) Bupropion
(C) Lithium
(D) Risperidone
(E) Topiramate
44. A 32-year-old nulligravid woman comes to the office because she has been unable to conceive during the past 12 months. She
and her husband have unprotected sexual intercourse three times weekly. Two years ago, she was involved in a motor vehicle
collision; CT scan at that time showed absence of the left kidney. She has no other history of serious illness, and her only
medication is a prenatal vitamin. She has no known allergies. Vital signs are within normal limits. Pelvic examination shows no
abnormalities. Hysterosalpingography is shown. When this patient conceives, she is at greatest risk for which of the following
pregnancy complications?
(A) Fetal macrosomia
(B) Multiple gestation
(C) Oligohydramnios
(D) Preeclampsia
(E) Preterm labor
31
45. Patient Information
Age: 14 years
Gender: F, self-identified
Ethnicity: unspecified
Site of Care: office
History
Reason for Visit/Chief Concern: "I feel light-headed."
History of Present Illness:
• 1-week history of episodes of light-headedness
• almost lost consciousness during three episodes
• experienced tunnel vision and almost fell when getting out of bed this morning
• evaluated 10 days ago for follow-up of attention-deficit/hyperactivity disorder; clonidine therapy initiated at that time
• sister is currently hospitalized for meningitis; patient has been receiving prophylactic rifampin
Past Medical History:
• moderate persistent asthma that is well controlled
• last asthma exacerbation was 1 week ago; 5-day course of prednisone was initiated at that time, which she completed
• attention-deficit/hyperactivity disorder diagnosed at age 8 years; has been on numerous different medications
Medications:
• albuterol
• fluticasone
• methylphenidate
• clonidine
• rifampin
Vaccinations:
• up-to-date
Allergies:
• no known drug allergies
Family History:
• mother and father: alive with no chronic conditions
• sister: currently being treated for meningitis
Psychosocial History:
• lives with parents, 18-year-old sister, and 2-year-old sister
• has difficulty in school because of attention-deficit/hyperactivity disorder
• does not smoke cigarettes, drink alcoholic beverages, or use nonprescribed medications or other substances
• not sexually active
Physical Examination
Temp
Pulse
Resp
BP
O
2
Sat
Ht
Wt
BMI
37.0°C
90/min
20/min
115/70 mm Hg
99%
155 cm (5 ft 1 in)
42 kg (93 lb)
17.5 kg/m
2
(98.6°F)
(lying down)
on RA
25th %ile
10th %ile
25th %ile
95/55 mm Hg
(after standing for 5 min)
• Appearance: sitting in a chair; comfortable, not in acute distress
• Skin: no rashes
• Pulmonary: clear to auscultation
• Cardiac: regular rhythm; no murmurs; capillary refill time of <2 seconds
• Abdominal: nondistended; normoactive bowel sounds; soft, nontender
• Neurologic: alert; cranial nerves intact
Question: An adverse effect of which of the following medications is the most likely cause of this patient's current condition?
(A) Albuterol
(B) Clonidine
(C) Methylphenidate
(D) Prednisone
(E) Rifampin
32
46. A 51-year-old woman comes to the office for a health maintenance examination. She feels well. She has no history of serious
illness and takes no medications. She smoked one pack of cigarettes daily for 28 years but quit 6 years ago. Pulse is 80/min,
respirations are 12/min, and blood pressure is 122/78 mm Hg. On examination, pedal pulses are absent bilaterally. A femoral
bruit is heard in the left lower extremity. Which of the following is the most appropriate next step in management?
(A) Pentoxifylline therapy
(B) Peripheral artery catheterization
(C) Serum lipid studies
(D) Warfarin therapy
(E) No further management is indicated
47. A 47-year-old woman comes to the office for a follow-up examination. Three days ago, she was brought to the emergency
department because of cellulitis of her right lower extremity, and trimethoprim-sulfamethoxazole therapy was begun. She has
had no adverse reaction to the medication. She has kept her leg elevated, and her cellulitis has improved moderately. She also
has hypertension, gout, and congenital heart disease. Her other medications are lisinopril, allopurinol, and warfarin. Five years
ago, she underwent mechanical mitral valve replacement. Temperature today is 36.7°C (98.0°F), pulse is 72/min, and blood
pressure is 128/83 mm Hg. Examination shows a 4 × 5-cm, erythematous, warm rash over the right lower extremity. There is no
red streaking, vesicles, or purulence. There is no rash elsewhere. Pulses in the lower extremities are intact. Lungs are clear to
auscultation. On cardiac examination, a mechanical click is heard during S
1
. A grade 2/6 holosystolic murmur is heard best at the
apex. The remainder of the examination shows no abnormalities. Results of laboratory studies are shown:
Blood
Hematocrit
35%
WBC
13,000/mm
3
Platelet count
357,000/mm
3
PTT
38 sec
PT
57 sec (INR=5.2)
Which of the following actions is most likely to have prevented the near miss in this patient?
(A) Review for potential drug-drug interaction
(B) Dietary counseling
(C) Echocardiography
(D) Venous duplex ultrasonography of the right lower extremity
(E) Discontinuation of allopurinol
48. A 58-year-old woman comes to the office for a routine examination. She has type 2 diabetes mellitus. During the past 3 months,
her fingerstick blood glucose concentrations have ranged from 80 mg/dL to 230 mg/dL; previously, they ranged from 80 mg/dL
to 140 mg/dL. Medications are metformin and aspirin. During the past 3 months, she has exercised three times weekly; previously,
she exercised once weekly. She follows a balanced diet. Despite exercise and diet, she has not had weight loss. She is 157 cm (5
ft 2 in) tall and weighs 72 kg (160 lb); BMI is 29 kg/m
2
. Examination shows a 1 × 2-cm, erythematous, soft lesion over the
posterior plantar aspect of the left foot. Sensation to pinprick and touch is decreased over the ankles and feet. Hemoglobin A
1c
is
7%. Which of the following findings are most likely to be present in this patient?
Insulin Receptor
Glucose
Insulin
Responsiveness
(A)
Increased
increased
decreased
(B)
Increased
increased
normal
(C)
Increased
normal
decreased
(D)
Increased
normal
normal
(E)
Normal
normal
normal
33
49. A 57-year-old woman comes to the emergency department because of a 2-day history of fever, chills, and sore throat. She is on
day 6 of a 7-day course of trimethoprim-sulfamethoxazole for treatment of cystitis. She has been taking lisinopril nightly for 2
years for treatment of hypertension. She has no other history of serious illness and takes no other medications. Temperature is
38.6°C (101.4°F), pulse is 84/min, respirations are 16/min, and blood pressure is 118/76 mm Hg. Examination shows erythema
of the pharynx but no exudates. No other abnormalities are noted. Results of laboratory studies are shown:
Blood
Hematocrit
38%
WBC
2200/mm
3
Neutrophils, segmented
16%
Neutrophils, bands
4%
Lymphocytes
64%
Monocytes
16%
Platelet count
225,000/mm
3
Which of the following is the most likely explanation for these findings?
(A) Acute mononucleosis
(B) Acute myelogenous leukemia
(C) Adverse effect of trimethoprim-sulfamethoxazole
(D) Allergic reaction to lisinopril
(E) Myelofibrosis
(F) Sepsis syndrome
50. A 39-year-old man is brought to the emergency department because of a 3-hour history of moderate pain and swelling of his right
knee and ankle. His symptoms began after he scaled a wall during an obstacle course. He landed awkwardly on his right leg and
rolled his right ankle; his knee buckled. He limped to the finish line. He underwent reconstruction of a torn right anterior cruciate
ligament 15 years ago. He and his boyfriend are engaged to be married. He feels safe at home. He does not appear to be in distress.
Examination of the right knee shows mild tenderness to palpation of the midsagittal joint line. Adduction and abduction stress
testing is stable at 0 and 30 degrees of flexion. Range of motion is full. Lachman test is negative. There is moderate tenderness
to palpation of the distal aspect of the fibula. There are ecchymoses over the anterolateral aspect of the right ankle. The ankle is
swollen. There is no tenderness to palpation of the medial aspect of the ankle. Range of motion of the ankle is limited to 5 degrees
of dorsiflexion and 30 degrees of plantar flexion because of pain. Which of the following is the most appropriate next step in
diagnosis?
Knee
Ankle
(A)
X-ray
x-ray
(B)
X-ray
no diagnostic testing indicated
(C)
No diagnostic testing indicated
x-ray
(D)
No diagnostic testing indicated
no diagnostic testing indicated
51. A 13-year-old girl is brought to the office for evaluation of short stature. Since birth, she has been below the 3rd percentile for
height and at the 3rd percentile for weight. She has never had a menstrual period. There is no family history of short stature, but
her maternal uncle did not begin puberty until the age of 16 years. Her current height is consistent with that of a 9½-year-old girl.
Pulse is 82/min and blood pressure is 110/70 mm Hg. Sexual maturity rating is stage 1 for breast development and stage 2 for
pubic hair development. Which of the following is the most appropriate next step in diagnosis?
(A) Complete blood count and serum chemistry panel
(B) Measurement of urine free cortisol concentration
(C) X-rays of the left hand and wrist
(D) X-rays of the long bones and spine
(E) X-ray of the skull
34
52. A 20-year-old woman comes to the office because of a 1-year history of a lump on her left ear that appeared 2 months after she
had the ear pierced. Physical examination shows the findings in the photograph. Which of the following is the most appropriate
initial treatment for this patient?
(A) Corticosteroid injection
(B) Cryotherapy
(C) Laser ablation
(D) Needle aspiration
(E) Wide excision
35
53. A 42-year-old woman is brought to the emergency department by ambulance 20 minutes after a head-on motor vehicle collision
in which she was the restrained driver. On arrival, she is alert and reports chest and abdominal pain. Medical history is
unremarkable and she takes no medications. Temperature is 36.7°C (98.0°F), pulse is 120/min, respirations are 28/min, and blood
pressure is 130/80 mm Hg. Pulse oximetry on 4 L/min of oxygen via nasal cannula shows an oxygen saturation of 90%. Breath
sounds are decreased on the right. Trachea is in the midline position. Cardiac examination discloses no abnormalities. An abrasion
consistent in shape with a seat belt is present over the right upper abdominal quadrant; the area is tender to palpation. Radial and
dorsalis pedis pulses are normal bilaterally. Results of laboratory studies are shown:
Arterial blood gas analysis on oxygen at 4 L/min
PO
2
65 mm Hg
PCO
2
32 mm Hg
pH
7.38
Blood
Hemoglobin
10.0 g/dL
Chest x-ray is shown. In addition to intravenous analgesic therapy, which of the following is the most appropriate next step in
management?
(A) Bronchoscopy
(B) Intubation and mechanical ventilation
(C) Placement of a thoracic epidural
(D) Thoracentesis
(E) Thoracotomy
(F) Tube thoracostomy
(G) Observation only
36
54. A 31-year-old woman comes to the office because of a 3-month history of intermittent nonbloody diarrhea. The first episode
began 5 days after a camping trip in rural Virginia with her family. During the trip, she drank from a spring-fed pond that she
says was “clean.” During the past 3 weeks, she also has had a 2.3-kg (5-lb) weight loss despite no change in appetite. She has not
had fever or other gastrointestinal symptoms. She has no history of serious illness and takes no medications. She is sexually active
with one female partner. The patient is 163 cm (5 ft 4 in) tall and weighs 54 kg (119 lb); BMI is 20 kg/m
2
. Temperature is 37.9°C
(100.2°F), pulse is 85/min and regular, respirations are 12/min, and blood pressure is 115/70 mm Hg. Abdominal examination
shows mild, diffuse tenderness to deep palpation; bowel sounds are increased. Test of the stool for occult blood is negative.
Results of a stool smear are shown. Which of the following is the most likely infectious agent?
(A) Campylobacter jejuni
(B) Clostridioides difficile
(C) Entamoeba histolytica
(D) Escherichia coli
(E) Giardia lamblia
(F) Salmonella enteritidis
(G) Shigella dysenteriae
37
55. A randomized placebo-controlled clinical trial is conducted to assess the effectiveness of a selective estrogen receptor modulator
(SERM) for chemoprophylaxis of breast cancer. A subset of the results shows:
SERM
Placebo
Number of participants
2400
1200
Woman-years of follow-up
7200
3600
Invasive breast cancer
7
13
Breast cancer rate (per 1000 woman-years)
1
3.6
Estrogen receptor status (invasive cancers)
Positive
2
10
Negative
4
2
Unknown
1
1
Which of the following represents the absolute risk reduction for invasive breast cancer per 1000 woman-years in those treated
with a SERM?
(A) 0.28
(B) 2.6
(C) 3.6
(D) 6
(E) 7
A 57-year-old woman comes to the emergency department 1 hour after vomiting bright red blood. She thinks it was approximately 1
cup of blood. She reports that during the past month, she has felt full sooner than usual while eating. She has not had other episodes of
vomiting and has not had weight loss. She has no history of operative procedures. She has hypertension and type 2 diabetes mellitus.
Medications are hydrochlorothiazide and metformin. She has not taken aspirin or nonsteroidal anti-inflammatory drugs during the past
month. She does not smoke cigarettes, drink alcohol, or use other substances. Temperature is 37.0°C (98.6°F), pulse is 116/min,
respirations are 16/min, and blood pressure is 110/78 mm Hg. Examination shows mild tenderness to palpation in the midepigastric
area; there is no rebound tenderness. Test of the stool for occult blood is positive. Hematocrit is 31%, platelet count is 240,000/mm
3
,
and prothrombin time is 13 seconds (INR=1.2).
56. In addition to intravenous administration of fluids and nasogastric tube lavage, which of the following is the most appropriate
next step in management?
(A) Abdominal CT scan
(B) Octreotide scan
(C) Technetium 99m scan
(D) Colonoscopy
(E) Esophagogastroduodenoscopy
(F) Mesenteric angiography
Items #56–57 are part of a sequential item set. In the actual examination environment, you will
not be able to view the second item until you click "Proceed to Next Item." After navigating to
the second item, you will not be able to add or change an answer to the first item.
38
57. After intravenous administration of fluids and nasogastric tube lavage, the patient undergoes esophagogastroduodenoscopy. A
photograph of the endoscopic findings is shown. Which of the following is the most appropriate next step in management?
(A) Octreotide therapy
(B) Omeprazole therapy
(C) Tyrosine kinase inhibitor therapy
(D) Endoscopic hemostatic therapy
(E) Endoscopic biopsy
(F) Endoscopic resection
END OF SET
58. A 77-year-old woman who resides in a skilled nursing care facility is examined by the physician 1 day after staff noticed some
blood on her bedsheets. She sustained a cerebral infarction 8 years ago and has been confined to bed since then because of residual
left hemiparesis. She is currently taking warfarin. She is 160 cm (5 ft 3 in) tall and weighs 50 kg (110 lb); BMI is 20 kg/m
2
.
Temperature is 36.0°C (96.8°F), pulse is 70/min, respirations are 18/min, and blood pressure is 180/90 mm Hg. There is no
lymphadenopathy. Examination shows a small, clean, decubitus ulcer on the sacrum. Pelvic examination shows a 3-cm, raised,
fleshy lesion on the right labium majus. Vaginal examination cannot be performed because of the patient's discomfort. Which of
the following is the most appropriate next step in management?
(A) Cytologic evaluation of the vulva
(B) CT scan of the abdomen and pelvis
(C) Application of an antifungal cream
(D) Application of a corticosteroid cream
(E) Colonoscopy
(F) Biopsy of the vulva
39
59. A 68-year-old man comes to the office because of a 3-month history of moderate low back pain that extends into both legs. He
first noticed the pain while standing in the ticket line at a theater. At that time, the pain was accompanied by tingling in both legs.
The symptoms recurred a few days later as he was standing in the grocery checkout line. The pain has not responded to ibuprofen
or naproxen but improves with sitting or lying down. Prior to onset of the pain, the patient typically walked for 30 minutes four
times weekly. He discontinued his walking regimen 2 months ago because it seemed to trigger his back pain. He has hypertension
and hyperlipidemia. Medications are hydrochlorothiazide-triamterene, simvastatin, and aspirin. The patient has smoked one pack
of cigarettes daily for 40 years. Pulse is 74/min and blood pressure is 140/90 mm Hg. Lungs are clear to auscultation; expiration
phase is delayed. On cardiac examination, a grade 2/6 systolic crescendo-decrescendo murmur is heard at the upper right sternal
border. There is trace ankle edema bilaterally; ankle brachial index is 1.0. Musculoskeletal and neurologic examinations show no
abnormalities. Which of the following is the most likely diagnosis?
(A) Abdominal aortic aneurysm
(B) Osteoporosis
(C) Peripheral neuropathy
(D) Spinal stenosis
(E) Vascular claudication
60. A 50-year-old man comes to the office because of a 3-day history of mild headache and intermittent confusion. He has a 30-year
history of schizophrenia and a 5-year history of emphysema. Medications are clozapine, sertraline, and inhaled beclomethasone
and salmeterol. He also receives home oxygen therapy. He smoked two packs of cigarettes daily for 33 years but quit 5 years
ago. Temperature is 36.8°C (98.2°F), respirations are 20/min, pulse is 90/min, and blood pressure is 115/70 mm Hg. Examination
shows moist mucous membranes. Lungs are clear to auscultation. There is no peripheral edema. Neurologic examination shows
no focal findings. Results of a complete blood count are within the reference ranges. Results of other laboratory studies are shown:
Serum
Na
+
116 mEq/L
K
+
4 mEq/L
Cl
90 mEq/L
Urea nitrogen
10 mg/dL
Creatinine
1 mg/dL
Glucose
100 mg/dL
Which of the following is the most appropriate next step in management?
(A) CT scan of the chest
(B) Measurement of serum cortisol and TSH concentrations
(C) Measurement of serum uric acid concentration
(D) Measurement of urine and plasma osmolarity
(E) MRI of the brain
61. A 67-year-old man is evaluated in the intensive care unit. He has end-stage pancreatic cancer and was hospitalized 3 days ago for
treatment of pneumonia. Respirations are 6/min. Pulse oximetry on 100% oxygen by face mask shows an oxygen saturation of
78%. Examination shows feeble respiratory efforts; he is using accessory muscles of respiration. On mental status examination,
the patient is oriented to person but not to place or time. If the patient is not endotracheally intubated and mechanically ventilated,
he will die within hours. His wife says the patient recently told her that he would never want mechanical ventilation, but they
never completed paperwork regarding his wishes. His daughter insists that he be mechanically ventilated. Which of the following
is the most appropriate action for the physician to take?
(A) Perform endotracheal intubation and begin mechanical ventilation
(B) Perform endotracheal intubation and then consult the hospital ethics committee regarding mechanical ventilation
(C) Perform endotracheal intubation only
(D) Provide palliative therapy only
(E) Seek a court order to assign a legal guardian
40
62. A 5-day-old boy is brought to the office for an initial well-child examination. He was born at 40 weeks' gestation and discharged
at 60 hours of life. On newborn screening, hemoglobin electrophoresis showed an FS pattern. He is at the 50th percentile for
length and weight. Temperature is 37.0°C (98.6°F), pulse is 136/min, and respirations are 34/min. He appears well. Examination
shows no abnormalities. Which of the following is the most appropriate next step in management?
(A) Deferoxamine therapy
(B) Hydroxyurea therapy
(C) Iron supplementation
(D) Monthly blood transfusions
(E) Penicillin prophylaxis
(F) Vitamin B
12
(cyanocobalamin) supplementation
63. A 1-week-old female newborn is brought to the office for a follow-up examination after newborn screening showed a serum TSH
concentration of 40 μU/mL (N=120). She has been breast-feeding well. She passes five to six stools and has multiple wet diapers
daily. She has returned to her birth weight. Vital signs are within normal limits. Examination discloses no abnormalities. Serum
studies today show a TSH concentration of 14 μU/mL (N=0.56.5) and free thyroxine (FT
4
) concentration of 0.4 ng/dL (N=0.9
2.2). Which of the following is the most appropriate next step in management?
(A) Hydrocortisone therapy
(B) Levothyroxine therapy
(C) Monthly serial measurement of serum thyroglobulin concentration
(D) Monthly serial measurement of serum TSH and free thyroxine (FT
4
) concentrations
(E) Radioactive iodine uptake scan
(F) Ultrasonography of the thyroid gland
64. An 18-year-old patient comes to the emergency department because of a 2-day history of severe abdominal pain. The pain began
in the middle of the abdomen after a large meal but now is localized to the right lower quadrant. The patient also has a 2-month
history of intermittent diarrhea. Three weeks ago, the patient had an upper respiratory tract infection that resolved spontaneously.
The patient currently takes no medications. The patient identifies as nonbinary and uses they/they pronouns; their assigned sex
at birth was male. They do not smoke cigarettes or use other substances. Temperature is 37.8°C (100.0°F), pulse is 80/min,
respirations are 20/min, and blood pressure is 110/70 mm Hg. Abdominal examination shows fullness and tenderness to palpation
of the right lower quadrant. Leukocyte count is 11,000/mm
3
. Which of the following is the most appropriate next step in
diagnosis?
(A) Abdominal ultrasonography of the right lower quadrant
(B) Air-contrast barium enema
(C) CT scan of the abdomen
(D) Technetium 99m scan of the bowel
(E) Upper gastrointestinal series with small bowel follow-through
65. A 25-year-old woman, gravida 3, para 2, at 36 weeks’ gestation comes to the emergency department because of heavy vaginal
bleeding following sexual intercourse 3 hours ago. She has received no prenatal care. She has no history of serious illness or
operative procedures. Vital signs are within normal limits. Continuous external fetal heart monitoring shows a baseline of 155/min
with no accelerations or decelerations. The uterus is consistent in size with a 36-week gestation. Tocodynamometer monitoring
shows contractions every 8 minutes. Which of the following is the most appropriate next step in management?
(A) Cesarean delivery now
(B) Digital examination of the cervix
(C) Intravenous administration of magnesium sulfate
(D) Intravenous administration of oxytocin
(E) Transabdominal ultrasonography
41
66. A 75-year-old man is admitted to the hospital because of a 3-day history of fever. He has type 2 diabetes mellitus, hypertension,
and chronic obstructive pulmonary disease. One year ago, he had a stroke with residual right lower extremity hemiparesis.
Medications are amlodipine, tiotropium, metformin, glimepiride, lisinopril, and aspirin. He does not drink alcoholic beverages
and has smoked one pack of cigarettes daily for 50 years. The patient is widowed, lives alone, and has three adult children. He is
not in acute distress. He is 178 cm (5 ft 10 in) tall and weighs 82 kg (180 lb); BMI is 26 kg/m
2
. Temperature is 39.2°C (102.6°F),
pulse is 110/min, respirations are 22/min, and blood pressure is 150/90 mm Hg. Pulse oximetry on room air shows an oxygen
saturation of 96%. Scattered end-expiratory wheezes are heard. There is dense hemiparesis of the right lower extremity.
Appropriate therapy is initiated, and 3 days later, the patient's condition has improved. He is awake, alert, and conversing with
one of his sons about current events. Temperature is 37.2°C (99.0°F), pulse is 72/min, and blood pressure is 130/82 mm Hg.
Discharge plans are discussed with the patient, his son, the physician, and a physical therapist. The physical therapist says the
patient would be best cared for in a nursing care facility. The patient acknowledges that a nursing facility would provide better
care for him but says he would prefer to remain at home with a visiting nurse. Once outside the patient's room, his son says, "I
have his power of attorney. Don't listen to him. He needs to be in a nursing home." Which of the following is the most appropriate
next step?
(A) Inform the son that the decision is the patient's to make, regardless of the medical team's recommendation
(B) Reassure the son that the patient will be admitted to a nursing care facility in accordance with the physical therapist's
recommendation
(C) Refrain from making any decisions until a family meeting with the son and siblings is arranged to discuss the matter
(D) Request psychiatric evaluation of the patient's mental competency
(E) Request to review the legal documentation designating the son as having power of attorney
67. A 59-year-old man is brought to the emergency department because of a 1-hour history of weakness of his left arm and leg and
mild headache. He says that, when he was gardening with his wife, he suddenly became unable to hold his gardening tool, lift his
left arm, or walk. Two weeks ago, he had a nonproductive cough and nasal congestion that resolved spontaneously after 1 week.
He has hypertension treated with lisinopril. On arrival, he is lethargic. He is 183 cm (6 ft) tall and weighs 97 kg (215 lb); BMI is
29 kg/m
2
. Temperature is 36.6°C (97.9°F), pulse is 94/min, respirations are 16/min, and blood pressure is 174/109 mm Hg. The
pupils measure 3 mm and are reactive to light. Muscle movement is decreased over the left lower aspect of the face. Muscle
strength is 0/5 in the left extremities. Deep tendon reflexes are absent in the left extremities and 2+ in the right extremities. CT
scan of the head shows a large area of increased attenuation in the right putamen region. Which of the following is the most likely
cause of this patient’s condition?
(A) Amyloid angiopathy
(B) Embolism
(C) Small vessel disease
(D) Vasculitis
(E) Venous thrombosis
68. A 24-year-old primigravid woman at 10 weeks’ gestation comes to the office for a scheduled dilatation and suction curettage.
Ultrasonography 2 days ago showed a fetus consistent in size with a 9-week gestation with absent cardiac activity. Two weeks
ago, the patient had an upper respiratory tract infection that resolved spontaneously. One year ago, she was diagnosed with herpes
simplex virus (HSV) 2; her last outbreak was 6 months ago. She has no other history of serious illness. She is 163 cm (5 ft 4 in)
tall. She weighed 82 kg (180 lb) prior to her pregnancy; BMI was 31 kg/m
2
. She has had a 3.6-kg (8-lb) weight gain during her
pregnancy. Vital signs are within normal limits. The vulvar area is shaven; no lesions are noted. Examination shows a normal-
appearing cervix. The uterus is nontender and consistent in size with an 8-week gestation. There is homogenous white-gray
vaginal discharge; the pH of the discharge is 5.5. She undergoes dilatation and suction curettage with sterile instruments and
povidone-iodine vaginal cleansing. Which of the following is the strongest predisposing risk factor for postoperative infection in
this patient?
(A) History of upper respiratory tract infection
(B) HSV 2
(C) Obesity
(D) Type of vaginal discharge
(E) Vulvar shaving
42
69. A 74-year-old woman comes to the clinic 2 days after the sudden onset of severe left foot pain. She has had difficulty walking
during this time and has been using a cane. She has not sustained trauma to the foot. She has type 2 diabetes mellitus and
hypertension. Medications are metformin, insulin glargine, hydrochlorothiazide, lisinopril, and atorvastatin. She typically wears
support stockings because of lower leg swelling, but the foot pain has made it difficult for her to wear them recently. She is a
traveling pastor and spends more than 4 hours daily in her car. She has been monogamous with her wife for the past 30 years.
Blood pressure is 146/88 mm Hg; other vital signs are within normal limits. A photograph of the left great toe is shown. No other
abnormalities are noted. Hemoglobin A
1c
is 7.4%, serum creatinine concentration is 1.4 mg/dL, and serum uric acid concentration
is 7.8 mg/dL. Results of serum electrolyte concentrations are within the reference ranges. Which of the following is the most
likely diagnosis?
(A) Cellulitis
(B) Gout
(C) Infected mucoid cyst
(D) Osteoarthritis
(E) Pseudogout
(F) Rheumatoid arthritis
70. A 47-year-old man comes to the office for a routine examination before beginning an exercise program. He feels well. He has a
25-year history of type 1 diabetes mellitus. A grade 1/6, early peaking, systolic ejection murmur is heard best at the left third
intercostal space. Sensation to light touch is mildly decreased over the toes. The remainder of the examination shows no
abnormalities. An exercise stress test shows a 3-mm ST-segment depression early in the test. The test does not produce any pain.
Which of the following is the most likely diagnosis?
(A) Cardiomyopathy
(B) Congenital heart disease
(C) Coronary artery disease
(D) Valvular heart disease
(E) Normal cardiac findings
43
71. A 22-year-old man comes to the office for a health maintenance examination. He feels well and has not noticed any health issues.
He is in the US Army. Two months ago, he returned from a 10-month deployment to Southwest Asia, during which he started
smoking one pack of cigarettes daily. The patient says he is happy to be home and has been celebrating with his friends and
family. He has been consuming an average of six beers daily since returning to the United States. He has no history of major
medical illness. He does not use any medications or other substances. His mother has type 2 diabetes mellitus. The patient is 180
cm (5 ft 11 in) tall and weighs 72 kg (160 lb); BMI is 22 kg/m
2
. His first blood pressure reading is 156/95 mm Hg; repeat reading
is 160/92 mm Hg. He says he remembers his blood pressure being normal when he had it checked in the past. Physical
examination discloses no other abnormalities. Which of the following is the most likely cause of this patient's increased blood
pressure?
(A) Cigarette smoking
(B) Essential hypertension
(C) Excessive alcohol use
(D) Pheochromocytoma
(E) Renal artery stenosis
72. A 32-year-old man comes to the office because of a 3-month history of a painless rash over his arms. During the past month, he
also has had a 3.2-kg (7-lb) weight loss despite no change in appetite. He has no history of serious illness and takes no medications.
He is 175 cm (5 ft 9 in) tall and weighs 72 kg (158 lb); BMI is 23 kg/m
2
. Pulse is 98/min; other vital signs are within normal
limits. Lungs are clear to auscultation. Cardiac examination shows no abnormalities. A photograph of the left upper extremity is
shown. Similar findings are noted over the right upper extremity. Which of the following is the most likely cause of this patient's
dermatologic findings?
(A) Actinic keratosis
(B) Dermatitis herpetiformis
(C) Kaposi sarcoma
(D) Lichen planus
(E) Seborrheic dermatitis
44
73. A 42-year-old nulligravid woman comes to the office because of a 6-month history of hot flashes and intermittent palpitations.
Her last menstrual period was 6 months ago. Menses previously occurred at regular 29-day intervals but gradually had occurred
less frequently during the past 2 years. She underwent ovarian cystectomy 13 years ago for a small benign teratoma. She used an
oral contraceptive for 12 years until tubal ligation 6 years ago. Her mother and sister had menopause at the ages of 47 and 49
years, respectively. The patient has smoked one pack of cigarettes daily for 26 years. She is sexually active and monogamous
with one male partner. She is 170 cm (5 ft 7 in) tall and weighs 86 kg (191 lb); BMI is 30 kg/m
2
. Pulse is 82/min and blood
pressure is 130/85 mm Hg. Physical examination, including pelvic examination, discloses no abnormalities. Which of the
following is the most appropriate next step in diagnosis?
(A) Measurement of serum follicle-stimulating hormone concentration
(B) Measurement of serum luteinizing hormone concentration
(C) Measurement of serum testosterone concentration
(D) ECG
(E) Pelvic ultrasonography
74. A colleague of a 47-year-old physician who is conducting patient evaluations during afternoon walk-in clinic hours notices that
the physician appears groggy, has slurred speech, and smells of alcohol. Earlier in the day, the physician had attended a
celebratory lunch for the graduation of one of the clinic's employees from nursing school. The colleague has not witnessed similar
findings in the physician in the past. The physician is scheduled to work at the clinic for the next 4 hours. A physician assistant
is present at the clinic and typically aids the physician with evaluating patients. Which of the following is the most appropriate
action for the colleague to take at this time?
(A) Allow the physician to finish his time at the clinic, but raise concern about the episode at the next physicians' staff
meeting
(B) Ask the office administrator to cancel the remainder of the clinic hours
(C) Ask the physician assistant to examine the physician's patients and discharge them before the physician has a chance to
examine them
(D) Confront the physician and ask him to leave the clinic
(E) Report the physician to the clinic administrator and conduct the remainder of the clinic hours without him
(F) Report the physician to the state medical board
75. A 68-year-old woman returns to the office for a follow-up examination. One month ago, urinalysis showed protein during work-
up of a suspected urinary tract infection. At that time, the patient was prescribed a 3-day course of oral cephalexin. Today, she
says her previous pain on urination and urinary frequency have resolved. She has major depressive disorder and osteoarthritis.
Current medications are venlafaxine and acetaminophen. Temperature is 37.0°C (98.6°F), pulse is 66/min, respirations are
18/min, and blood pressure is 122/66 mm Hg. The patient appears physically fit. Examination discloses no abnormalities. Repeat
urinalysis is shown:
Specific gravity
1.015 (N=1.003–1.029)
pH
6.0 (N=4.57.8)
Protein
negative
Blood
negative
Leukocyte esterase
negative
Nitrite
negative
WBCs
3/hpf
RBCs
1/hpf
Which of the following is the most appropriate additional diagnostic study at this time?
(A) Determination of urine albumin:creatinine ratio
(B) 24-Hour urine collection for measurement of protein concentration
(C) Ultrasonography of the kidneys
(D) Urine sulfosalicylic acid method
(E) No additional testing is indicated
45
76. A 22-year-old man is brought to the emergency department because of severe right lower leg pain since he fell in the woods
during a camping trip 2 days ago. The pain has worsened during the past 12 hours. He sustained a small scratch below his knee
as a result of the fall. He has no history of serious illness and uses no medications or other substances. He does not drink alcohol.
Pulse is 110/min, respirations are 14/min, and blood pressure is 105/60 mm Hg. Examination of the right lower extremity shows
edema, diffuse cellulitis, and hemorrhagic bullae from the knee to the ankle. No abnormalities of the left lower extremity are
noted. Peripheral intravenous antibiotic therapy and infusion of 2 L of crystalloid fluid are begun. Which of the following is the
most appropriate next step in management?
(A) Fine-needle aspiration of the bullae
(B) Hyperbaric oxygen therapy
(C) MRI of the right lower extremity
(D) Surgical debridement
(E) Hospital admission for observation only
77. A 27-year-old woman, gravida 3, para 2, at 12 weeks' gestation comes to the office for her first prenatal visit. Her first child was
delivered at 41 weeks' gestation and weighed 4167 g (9 lb 3 oz). Her second child was delivered at 37 weeks' gestation and
weighed 4309 g (9 lb 8 oz). She has no history of serious illness and her only medication is a prenatal vitamin. Her sister has a
history of delivering a stillborn infant. Her mother and father both have type 2 diabetes mellitus. Eight months ago, the patient
immigrated to the United States from Mexico. The patient is 157 cm (5 ft 2 in) tall. She weighed 95 kg (210 lb) prior to her
pregnancy; BMI was 38 kg/m
2
. She has had a 3-kg (6.6-lb) weight gain during her pregnancy. Pulse is 80/min, respirations are
14/min, and blood pressure is 110/60 mm Hg. Bimanual examination shows a uterus consistent in size with a 12-week gestation.
Ultrasonography confirms a 12-week gestation. Which of the following is the most appropriate next step in management?
(A) Antiphospholipid antibody screening
(B) Chest x-ray
(C) Fasting glucose tolerance test
(D) Measurement of serum α-fetoprotein concentration
(E) Measurement of serum free thyroxine concentration
78. An 84-year-old woman comes to the office because of a 1-month history of malaise. She has not had fever, weight loss, or
abdominal pain. She has type 2 diabetes mellitus well controlled with long-acting insulin and metformin. She has missed multiple
appointments during the past 6 months with no reasons given. Her hemoglobin A
1c
6 months ago was 6.7%. She says she has
been “feeling fine.” Her son moved back into her home 9 months ago because he had lost his job. The patient’s husband died 5
years ago, and she has no other children. She appears unkempt. She is alert and fully oriented. She is 163 cm (5 ft 4 in) tall and
weighs 77 kg (170 lb); BMI is 29 kg/m
2
. Her pulse is 80/min and regular, respirations are 18/min, and blood pressure is 160/94
mm Hg. Mucous membranes are dry; there is no thrush. The skin is dry and intact. The remainder of the physical examination
shows no abnormalities. Mental status examination shows no memory deficit. She has a normal mood and flat affect. Her fasting
serum glucose concentration is 380 mg/dL. Results of urinalysis are shown:
Glucose
4+
Ketones
none
Nitrites
none
Leukocyte esterase
none
Which of the following is the most likely diagnosis?
(A) Dietary indiscretion
(B) Elder neglect
(C) Major depressive disorder
(D) Pancreatic malignancy
(E) Urinary tract infection
46
79. A 15-year-old boy is brought to the office 2 hours after he awoke with weakness of the right side of his face. He has a 2-week
history of intermittent headache, sensitivity to light, and mild to moderate neck pain. He has no history of serious illness. His
only medication is acetaminophen as needed. He lives in New Jersey and has not traveled outside of the state during the past 6
months. Temperature is 37.4°C (99.3°F), pulse is 78/min and regular, and blood pressure is 108/70 mm Hg. Flexion of the neck
elicits moderate pain and resistance. The right eyelids do not close completely. Ocular movements are normal. There is no
evidence of ptosis. The right side of the forehead does not wrinkle when the patient looks up. There is severe weakness of the
right corner of the mouth. There are no abnormalities of the left side of the face. Muscle strength in the jaw, tongue, and palate
is normal and symmetric. There is no dysarthria. Muscle strength is 5/5 in all extremities. Coordination and sensation are intact.
Gait is normal. A lumbar puncture is performed, and opening pressure is 180 mm H
2
O. Cerebrospinal fluid analysis results are
shown:
Glucose
60 mg/dL
Total protein
52 mg/dL
WBC
85/mm
3
Segmented neutrophils
30%
Lymphocytes
70%
RBC
1/mm
3
Which of the following is the most appropriate next step in diagnosis?
(A) Measurement of serum angiotensin-converting enzyme activity
(B) Measurement of serum Lyme (Borrelia burgdorferi) antibody concentration
(C) Polymerase chain reaction test for cytomegalovirus
(D) Serum antinuclear antibody assay
(E) Serum protein electrophoresis
80. A 2-year-old girl is brought to the emergency department because of loud breathing, harsh cough, and a hoarse cry since she
awoke 1 hour ago. Her parents state that yesterday she had a runny nose. Medical history is unremarkable and she receives no
medications. Temperature is 38.5°C (101.3°F), pulse is 120/min, respirations are 32/min, and blood pressure is 105/65 mm Hg.
There are moderate suprasternal and subcostal retractions. Inspiratory stridor is heard at rest. Which of the following is the most
appropriate initial pharmacotherapy?
(A) Intravenous dexamethasone
(B) Nebulized albuterol
(C) Nebulized budesonide
(D) Nebulized epinephrine
(E) Oral albuterol
(F) Oral prednisone
(G) Subcutaneous epinephrine
47
BLOCK 3, ITEMS 81–120
81. A 28-year-old man is admitted to the intensive care unit 3 hours after undergoing resection and anastomosis of a small-bowel
injury and nephrectomy because of a right renal laceration sustained in a motor vehicle collision. He also has bilateral, minimally
displaced pubic rami fractures. He is intubated and mechanically ventilated. He is receiving morphine and propofol. He responds
only to painful stimuli. Pulse is 110/min, ventilatory rate is 12/min, and blood pressure is 133/68 mm Hg. The abdomen is
distended and soft, and the incision is clean, dry, and intact. Since the operation, urine output had been 40 mL/h and bloody.
During the past hour, urine output has been 5 mL and bloody. In addition to intravenous administration of 0.9% saline, which of
the following is the most appropriate next step in management?
(A) CT scan of the abdomen and pelvis
(B) Repeat surgical exploration of the abdomen
(C) Transfusion of packed red blood cells
(D) Ultrasonography of the bladder
82. A 15-year-old girl is brought to the office because of a 3-day history of diffuse abdominal pain, fever, and vomiting. She has
cerebral palsy, situs inversus, and severe scoliosis. She uses a wheelchair for ambulation. During previous visits, transferring this
patient from her wheelchair to the examination table has been time-consuming and has caused the patient discomfort. Today, the
patient appears uncomfortable. The physician decides to conduct the examination while the patient is sitting in her wheelchair.
Temperature is 38.3°C (101.0°F), pulse is 110/min, respirations are 20/min, and blood pressure is 110/50 mm Hg. Pulse oximetry
on room air shows an oxygen saturation of 100%. Examination shows mildly dry mucous membranes. The physician evaluated
four previous patients today who had vomiting and fever; he diagnosed gastroenteritis in all four patients. The physician attributes
this patient's increased pulse to mild dehydration and fever, and he diagnoses this patient with gastroenteritis. The physician
prescribes ondansetron and tells the patient to return if she has intractable vomiting or if her abdominal pain worsens. Five hours
later, the patient is taken to the emergency department after her mother finds her listless and barely responsive. Temperature is
38.9°C (102.0°F), pulse is 180/min, respirations are 35/min, and blood pressure is 70/50 mm Hg. Pulse oximetry on room air
shows an oxygen saturation of 96%. CT scan of the abdomen shows volvulus, and emergency resection of necrotic bowel is done.
Which of the following best describes the error that occurred?
(A) Latent error
(B) Near miss
(C) Non-preventable error
(D) Premature closure
(E) Systems failure
83. A 27-year-old man comes to the office because of a 1-month history of depressed mood and fatigue. He reports that it takes him
at least 1 hour to fall asleep at night. He has had decreased concentration at work and decreased interest in socializing with his
friends. He no longer exercises. He has epilepsy well controlled with levetiracetam. Physical examination discloses no
abnormalities. On mental status examination, he has a sad mood and is briefly tearful. He has no suicidal ideation, hallucinations,
or delusions. Which of the following is the most appropriate pharmacotherapy?
(A) Alprazolam
(B) Aripiprazole
(C) Bupropion
(D) Buspirone
(E) Mirtazapine
USMLE STEP 2 CK SAMPLE TEST QUESTIONS
48
84. A 23-year-old nulligravid woman comes to the office because she has been unable to conceive during the past year. She and her
husband have had regular unprotected intercourse during this time. The husband has no children. Prior to attempting to conceive,
the patient regularly used depot medroxyprogesterone. Menses occur at regular 28-day intervals with occasional mid-cycle
abdominal pain previously diagnosed as mittelschmerz. She had a ruptured appendix at age 17 years and pelvic inflammatory
disease at age 19 years. At age 21 years, cervical cytology showed a low-grade squamous intraepithelial lesion; repeat cervical
cytology 1 year later showed no abnormalities. Current medications are a prenatal vitamin and occasional ibuprofen for mid-
cycle abdominal pain. She is 168 cm (5 ft 6 in) tall and weighs 59 kg (130 lb); BMI is 21 kg/m
2
. Vital signs are within normal
limits. Physical examination discloses no abnormalities. Hysterosalpingography is done, and x-rays obtained during the procedure
are shown. Without treatment, which of the following is the most likely clinical course for this patient?
(A) Female factor infertility
(B) Hydatidiform mole
(C) Recurrent spontaneous abortions
(D) Successful pregnancy within the next year
85. A 50-year-old woman comes to the office for a health maintenance examination. She feels well. Medical history is unremarkable
and she takes no medications. There is no family history of serious illness. Physical examination, including breast examination,
discloses no abnormalities. Mammography shows a cluster of 20 pleomorphic microcalcifications in a 1-cm area at the 2-o'clock
position of the right breast. Which of the following is the most likely diagnosis?
(A) Ductal carcinoma in situ
(B) Fat necrosis
(C) Fibroadenoma
(D) Mastitis
(E) Sclerosing adenosis
49
86. A community has created standard-of-care guidelines for ambulatory patients with several common diagnoses. The community's
standard of care for patients with asthma is annual pulmonary function testing. Recent analysis of billing records from one clinic
in this community shows that only 34% of patients with asthma at the clinic undergo pulmonary function testing. Enacting which
of the following procedures is most likely to improve this clinic's adherence to the community's standard of care for patients with
asthma?
(A) Annual chart reviews of patients with asthma and feedback to the physicians
(B) Diagnosis-driven reminders in patient charts
(C) Placement of flyers in the clinic waiting room that remind patients with asthma to undergo annual pulmonary function
testing
(D) Pulmonary function testing of all patients examined for respiratory symptoms
(E) Pulmonary function testing of patients with asthma at each examination preceded by an exacerbation
87. A 27-year-old woman, gravida 1, para 1, comes to the emergency department because of a 1-day history of moderate pain in her
right breast. Three weeks ago, she underwent cesarean delivery of a healthy newborn at term because of breech presentation. She
is breast-feeding, but the newborn is not latching properly on the right. She has no history of serious illness. Medications are
acetaminophen for postoperative pain and a prenatal vitamin. Temperature is 38.9°C (102.0°F). Examination of the right breast
shows a 5 × 2-cm wedge-shaped area of erythema and moderate tenderness. The left breast is normal. Abdominal examination
shows a well-healing surgical incision. In addition to beginning antipyretic therapy, which of the following is the most appropriate
next step in management?
(A) Application of a breast binder on the right
(B) Dicloxacillin therapy
(C) Fluconazole therapy
(D) Fine-needle aspiration of the erythematous area
(E) No further management is indicated
88. A 12-year-old girl is brought to the office for a well-child examination. She feels well. She has no history of serious illness and
receives no medications. Menarche has not yet occurred. She is at the 30th percentile for height and 60th percentile for weight.
Examination of the breasts shows no glandular tissue; the areolae follow the skin contours of the chest. There is no pubic hair.
The remainder of the examination shows no abnormalities. Which of the following is the most appropriate next step in diagnosis?
(A) Measurement of serum follicle-stimulating hormone and luteinizing hormone concentrations
(B) Measurement of serum growth hormone and thyroxine concentrations
(C) MRI of the brain
(D) X-ray of the left hand and wrist to determine bone age
(E) No additional diagnostic steps are indicated
89. A 42-year-old woman comes to the office because of a 2-month history of episodes of light-headedness and loss of consciousness
for 10 seconds. The episodes occur two to three times weekly at work. She says her light-headedness resolves after she eats
snacks. During this time, she also has had a 9-kg (20-lb) weight gain. She has no history of serious illness and takes no
medications. She is 160 cm (5 ft 3 in) tall and weighs 72 kg (160 lb); BMI is 28 kg/m
2
. Vital signs are within normal limits.
Physical examination, including neurologic examination, shows no abnormalities. Serum glucose concentration is 41 mg/dL,
serum C peptide concentration is 0.5 ng/mL (N=0.52.5), and serum insulin concentration is 80 μIU/mL (N=520). CT scan of
the abdomen is most likely to show which of the following findings?
(A) Duodenal mass with multiple liver metastases
(B) Fluid-filled mass between the posterior aspect of the stomach and pancreas
(C) Low-density mass in the head of the pancreas obstructing the main duct
(D) Vascular mass in the neck of the pancreas
(E) No abnormalities
50
90. A hospital with a large intensive care unit (ICU) would like to improve communication among team members who care for
patients with complex conditions. Research has shown that team communication is most effective when one clinician is
designated as the team leader and assumes responsibility for directing the care. The team leader ensures that all team members
share a common understanding of the patient and openly discuss their views about the case. The common understanding includes
the patient’s diagnosis, prognosis, and care plan. This allows the team to quickly recognize when new data deviate from the
expected and then reassess the team's approach to the patient. Which of the following is the most appropriate method for
maintaining a common mental model among all team members?
(A) Conduct weekly structured team briefings and daily huddles
(B) Educate all team members on protocols for common illnesses
(C) Have the team leader discuss the case with each team member individually on a regular basis
(D) Maintain a consistent team of intensivists in the ICU
(E) Maintain a disease-specific ICU
91. A 12-month-old girl is brought to the office by her father because of a 3-day history of fever and mildly decreased appetite and
activity. She has not had cough, vomiting, or diarrhea. She receives no medications. Vaccinations are up-to-date. She is at the
35th percentile for height, 50th percentile for weight, and 60th percentile for head circumference. Temperature is 39.2°C
(102.6°F); other vital signs are within normal limits. Examination shows no other abnormalities. A catheter urine sample is
obtained, and urinalysis shows 2030 WBC/hpf and is positive for nitrites. Oral trimethoprim-sulfamethoxazole therapy is begun.
The next day, urine culture grows greater than 100,000 colonies/mL of Escherichia coli that is susceptible to trimethoprim-
sulfamethoxazole. Which of the following is the most appropriate next step in management?
(A) Ciprofloxacin prophylaxis
(B) Clindamycin prophylaxis
(C) CT urography
(D) Ultrasonography of the kidneys and bladder
(E) Voiding cystourethrography
92. A 32-year-old man comes to the office because of a 2-week history of persistent nonproductive cough occurring in short bursts.
During the past week, he has had bilateral chest pain with severe coughing. He has vomited several times after coughing
vigorously. He has not had shortness of breath or fever. Three weeks ago, he had a runny nose, sneezing, and mild malaise that
resolved spontaneously after 1 week. Toward the end of that period, he noticed the onset of cough, which has since worsened.
He has no history of serious illness, and his only medication is over-the-counter cough drops. He drinks 24 to 36 oz of beer
weekly. He does not smoke cigarettes or use other substances. He is sexually active with one male partner; they use condoms
consistently. He is 173 cm (5 ft 8 in) tall and weighs 86 kg (190 lb); BMI is 29 kg/m
2
. Temperature is 37.0°C (98.6°F), pulse is
70/min, respirations are 12/min, and blood pressure is 120/80 mm Hg. Pulse oximetry on room air shows an oxygen saturation
of 98%. During the examination, the patient coughs several times. There is no sinus tenderness to palpation and no cervical
lymphadenopathy. The oropharynx appears normal. Lungs are clear to auscultation. Cardiac examination discloses no
abnormalities. Which of the following is the most appropriate pharmacotherapy?
(A) Albuterol
(B) Azithromycin
(C) Levofloxacin
(D) Omeprazole
(E) Prednisone
51
93. Patient Information
Age: 18 years
Gender: F, self-identified
Race/Ethnicity: unspecified
Site of Care: emergency department
History
Reason for Visit/Chief Concern: "I'm coughing up green and yellow mucus."
History of Present Illness:
• cystic fibrosis with two to three exacerbations yearly for past 5 years
• 3-day history of cough productive of copious amounts of yellow and green sputum
• associated with subjective fever, shaking chills, and progressive shortness of breath
• symptoms occurred first only with exertion but now occur at rest
• also reports sharp left-sided chest pain that she attributes to extended paroxysms of coughing
• symptoms are similar to previous exacerbations
Past Medical History:
• cystic fibrosis complicated by bronchiectasis and pancreatic insufficiency
Medications:
• pancrelipase
• dornase alfa
• ivacaftor
• albuterol prn for pulmonary symptoms
• 3.0% inhaled saline prn for pulmonary symptoms
Allergies:
• no known drug allergies
Family History:
• brother: age 13 years with cystic fibrosis
Psychosocial History:
• does not smoke cigarettes, drink alcoholic beverages, or use other substances
• high school student
Physical Examination
Temp
Pulse
Resp
BP
O
2
Sat
Ht
Wt
BMI
37.9°C
112/min
22/min
138/74 mm Hg
92%
157 cm
54 kg
22 kg/m
2
(100.2°F)
and regular
on RA
(5 ft 2 in)
(118 lb)
• Appearance: in mild respiratory distress; using accessory muscles of respiration; frequent coughing productive of yellow and
green sputum
• Pulmonary: bilateral diffuse crackles with moderate diffuse expiratory wheezes more prominent in the apices
Diagnostic Studies
Blood
Hematocrit
42%
Hemoglobin
13.1 g/dL
WBC
16,400/mm
3
Platelet count
334,000/mm
3
• portable chest x-ray: bilateral basilar consolidations with increased interstitial markings; no effusions or pneumothorax
Question: In addition to initiating broad-spectrum antibiotic therapy, which of the following is the most appropriate next step in
evaluation?
(A) CT angiography of the chest
(B) Serum immunoglobulin concentrations
(C) Spirometry with lung volume measurement
(D) Sputum culture
(E) Transbronchial lung biopsy
52
94. A 16-year-old boy comes to the emergency department (ED) because of moderate left knee pain and abrasions sustained 2 hours
ago in a skateboarding collision. He has a history of occasional injuries sustained while skateboarding, including an injury to the
same knee 6 months ago. He says his knee had felt fine until today's injury, and he has been fit and active. The patient has an
EXT1 mutation, which is also present in his father. The patient takes no medications. Vital signs are within normal limits.
Examination shows superficial abrasions over the anterolateral aspect of the left knee. No effusion or instability is noted in the
knee, but there is tenderness to palpation. The patient walks with a limp but is able to bear weight on the leg. X-ray of the left
knee shows two 1 × 2-cm bone polyps extending, one medially and one laterally, from the proximal tibia at the level of the closing
physis; this is unchanged from an x-ray of the same knee obtained during his previous ED visit 6 months ago. Which of the
following is the most appropriate recommendation for activity and additional workup?
(A) Biopsy of the polyps and recommendation to bear weight as tolerated
(B) Biopsy of the polyps and recommendation for no weight bearing
(C) MRI of the knee and recommendation to bear weight as tolerated
(D) MRI of the knee and recommendation for no weight bearing
(E) Recommendation to bear weight as tolerated; no further workup needed
(F) Recommendation for no weight bearing; no further workup needed
95. A 70-year-old woman comes to the office because of three episodes of rectal bleeding during the past 4 days. During each episode,
she had the sudden urge to have a bowel movement and then passed a bright red, bloody stool. She has not had chest, abdominal,
or rectal pain, palpitations, dizziness, light-headedness, or black stools. Her most recent colonoscopy 8 years ago showed no
abnormalities. She has osteoarthritis treated with daily ibuprofen. She takes no other medications. She is 168 cm (5 ft 6 in) tall
and weighs 69 kg (152 lb); BMI is 25 kg/m
2
. Temperature is 37.0°C (98.6°F), and respirations are 16/min. While sitting, her
pulse is 80/min and blood pressure is 130/76 mm Hg. While standing, her pulse is 88/min and blood pressure is 126/74 mm Hg.
On examination, mucous membranes are moist. The abdomen is soft, nontender, and nondistended. There is no rebound
tenderness or guarding. Bowel sounds are normal. Rectal examination shows no stool in the vault; there are no masses. Which of
the following is the most likely diagnosis?
(A) Anal fissure
(B) Colonic polyp
(C) Diverticulosis
(D) Duodenal ulcer
(E) Gastritis
(F) Ulcerative colitis
96. A 14-year-old boy is brought to the office because of a 6-month history of changes in behavior. Two days ago, the patient and his
friends were detained by police for trespassing on abandoned property and entering an unoccupied house. No alcohol or other
substances were involved. The police officer who brought the patient home reported that there was evidence indicating this had
not been the first time the boys entered the property. The mother says her son was polite and close with his parents until 6 months
ago when he began socializing with a different group of boys in his class. She says the patient is constantly exchanging text
messages with his new friends. The group has inside jokes and nicknames for one another. Every weekend, the patient goes out
with his friends and argues with his parents about his curfew. At home, he is irritable and shares few details about his outside
activities with his parents. He has no history of serious illness and receives no medications. Examination shows no abnormalities.
Urine toxicology screening is negative. Which of the following is the most likely explanation for this patient’s behavior?
(A) Adjustment disorder
(B) Attention-deficit/hyperactivity disorder
(C) Conduct disorder
(D) Oppositional defiant disorder
(E) Normal development
53
97. A 6-year-old boy is brought to the emergency department by his mother because of the acute onset 1 hour ago of severe right-
sided lumbar pain and three episodes of emesis. The pain is sharp and causes him to double over and cry. Medical and family
histories are unremarkable. The patient takes no medications. Temperature is 36.7°C (98.0°F), pulse is 150/min, respirations are
30/min, and blood pressure is 110/70 mm Hg. He appears to be in severe pain. Abdominal examination discloses right upper
quadrant tenderness to palpation. The remainder of the examination discloses no abnormalities. Intravenous morphine is
administered and results in improvement in the patient's pain. Results of dipstick urinalysis are shown:
Specific gravity
1.035 (N=1.0031.029)
pH
5.5
Protein
30 mg/dL
Glucose
Negative
Bilirubin
Negative
Blood
4+
Leukocyte esterase
1+
Nitrite
Negative
Urobilinogen
Negative
WBCs
5–10/hpf
RBCs
50100/hpf
Renal ultrasonography shows severe right-sided hydronephrosis with moderate hydroureter to the bladder. Which of the following
is the most appropriate next step in evaluation?
(A) Captopril renography
(B) CT scan of the abdomen and pelvis
(C) MAG-3 renal scan with furosemide
(D) Radionuclide cystography
(E) Retrograde pyelography
(F) Ultrasonography of the spinal column
(G) Voiding cystourethrography
98. A 20-year-old man comes to the office at the end of the summer because of a 2-month history of generalized malaise, fatigue,
intermittent cough productive of whitish sputum, and decreased appetite. During this time, he has lost approximately 4 kg (10
lb). The patient previously was on the college track team but now feels winded when he tries to go running. He has a history of
childhood asthma. He currently takes no medications. He does not smoke cigarettes, drink alcoholic beverages, or use other
substances. He identifies as African American. He has been working at a lumber mill for the summer, feeding logs into a saw.
He is 178 cm (5 ft 10 in) tall and weighs 77 kg (170 lb); BMI is 24 kg/m
2
. Temperature is 37.7°C (99.9°F), pulse is 78/min,
respirations are 20/min, and blood pressure is 114/70 mm Hg. Diffuse crackles are heard bilaterally. The remainder of the
examination shows no abnormalities. Chest x-ray shows a reticulonodular pattern that is more prominent in the upper lung fields.
Results of pulmonary function tests show a mixed obstructive and restrictive pattern. Which of the following is the most likely
diagnosis?
(A) Aspergillosis
(B) Asthma recurrence
(C) Hypersensitivity pneumonitis
(D) Sarcoidosis
(E) Silicosis
54
99. A 32-year-old woman is admitted to the hospital because of a 2-year history of intractable seizures. Her boyfriend, who has
accompanied her, describes the seizures as episodes of bilateral limb shaking and moving her head from side to side, during
which the patient closes her eyes and cries. The episodes last 15 to 20 minutes, and she is intermittently responsive during this
time. Trials of phenytoin, carbamazepine, gabapentin, oxcarbazepine, and divalproex have not provided relief of her symptoms.
Her current medications are zonisamide and pregabalin. The patient has a history of childhood sexual and physical abuse. Vital
signs are within normal limits. Neurologic examination shows no focal findings. The patient undergoes long-term video EEG
monitoring, which shows that her seizures are not associated with any EEG changes. Which of the following is the most
appropriate next step in management?
(A) Cognitive behavioral therapy
(B) Hypnotic therapy
(C) Increasing the dosage of pregabalin
(D) Psychoanalytic therapy
(E) Surgical resection of the epileptogenic focus
100. A 2-year-old girl is brought to the clinic by her parents for an initial examination. Medical records show that she has a cleft palate
and ventriculoseptal defects that have not been repaired. During the past 18 months, she has had two episodes of pneumonia and
at least 10 episodes of otitis media treated with antibiotic therapy. The patient’s parents recently adopted her from China. She is
at the 10th percentile for length and 5th percentile for weight. Vital signs are within normal limits. Examination shows middle
ear effusions bilaterally and a cleft palate. Lungs are clear to auscultation. A grade 3/6 holosystolic murmur is heard on cardiac
examination. There is no hepatosplenomegaly. Chest x-ray is shown. A complete blood count is most likely to show which of the
following leukocyte findings in this patient?
(A) Eosinophilia
(B) Lymphocytosis
(C) Lymphopenia
(D) Neutropenia
(E) Neutrophilia
55
101. A 25-year-old primigravid woman at 28 weeks' gestation comes to the office for a routine prenatal visit. At 18 weeks' gestation,
she had vaginal bleeding that resolved spontaneously. At that time, her blood group was determined to be O, Rh-negative and
she received Rh
o
(D) immune globulin. She reports good fetal movement. Blood pressure is 120/70 mm Hg. Fetal heart rate is
160/min. Fundal height is 30 cm. The fetus is in a vertex presentation. Hemoglobin concentration is 11 g/dL and hematocrit is
34%. Serum antibody assay is positive; the anti-D titer is too weak to measure. Results of serologic testing for syphilis and HIV
antibody testing are negative. Which of the following is the most appropriate next step in management?
(A) Repeat serum anti-D antibody titer
(B) Ultrasonography of the pelvis
(C) Administration of Rh
o
(D) immune globulin
(D) Amniocentesis
(E) Induction of labor
102. A 5-year-old boy is brought to the office by his parents in July because of a 2-day history of an itchy rash. The rash began as small
red bumps on the right forearm and spread to the left cheek and right ankle within the same day. He has not had fever, vomiting,
or diarrhea. No other family members have similar symptoms. Medical history is unremarkable and he receives no medications.
The family switched to a new brand of bath soap last week. They have three pet cats and one dog. Three days ago, the family
attended a picnic at a nearby park. Temperature is 37.0°C (98.6°F), pulse is 90/min, and respirations are 20/min. The patient is
alert and scratching his right forearm. Examination shows mild edema of the left cheek. A photograph of the right forearm is
shown. Similar lesions are noted over the left cheek and right ankle. The remainder of the examination shows no abnormalities.
Avoidance of which of the following is most likely to have prevented this patient’s condition?
(A) Dog
(B) Flora
(C) Peanut butter
(D) Perfumed bath soap
(E) Sun
56
103. Patient Information
Age: 56 years
Gender: F, self-identified
Race/Ethnicity: mixed race (White and Honduran)
Site of Care: clinic
History
Reason for Visit/Chief Concern: "I'm tired and I don't feel well."
History of Present Illness:
• fatigue and malaise for 3 months; tires quickly with physical activity
• early satiety
Past Medical History:
• 5-year history of hypertension
• 5-year history of hyperlipidemia
Medications:
• chlorthalidone
• enalapril
• atorvastatin
Allergies:
• no known drug allergies
Physical Examination
Temp
Pulse
Resp
BP
O
2
Sat
Ht
Wt
BMI
37.5°C
75/min
14/min
139/78 mm Hg
162 cm
94 kg
36 kg/m
2
(99.5°F)
(5 ft 4 in)
(207 lb)
• HEENT: normal conjunctivae and oropharynx; thyroid gland is normal in size with no mass or tenderness
• Pulmonary: clear to auscultation
• Cardiac: S
1
and S
2
; no murmur or gallop
• Abdominal: nontender; liver edge palpated at the right costal margin with inspiration; dullness to percussion over the fifth to
eighth intercostal spaces at the left anterior axillary line
Diagnostic Studies
Blood
Urine
RBC
4.5 million/mm
3
Protein
3+
Hematocrit
39%
Hemoglobin
13.0 g/dL
MCV
86.7 μm
3
WBC
67,000/mm
3
Neutrophils, segmented
83%
Lymphocytes
2%
Eosinophils
5%
Basophils
4%
Metamyelocytes
3%
Myelocytes
3%
Question: Which of the following is the most likely diagnosis?
(A) Acute lymphocytic leukemia
(B) Acute myeloid leukemia
(C) Chronic lymphocytic leukemia
(D) Chronic myeloid leukemia
(E) Monoclonal gammopathy of undetermined significance
57
104. A 65-year-old woman comes to the office because of a 6-month history of progressive shortness of breath and a 1-year history of
nonproductive cough. Initially, the shortness of breath occurred only with exertion but now occurs at rest. It does not worsen
when she lies down but is still present. Her symptoms have limited her ability to travel internationally with the Peace Corps. She
has not had chest pain, fever, chills, or blood-tinged sputum. She has hypertension, type 2 diabetes mellitus, and major depressive
disorder. Medications are amlodipine, lisinopril, insulin glargine, and sertraline. She has smoked one pack of cigarettes daily for
50 years. She drinks one glass of wine on special occasions. Temperature is 37.2°C (99.0°F), pulse is 77/min, respirations are
16/min, and blood pressure is 132/82 mm Hg. Pulse oximetry on room air shows an oxygen saturation of 94%. There is no jugular
venous distention. On pulmonary examination, end-expiratory wheezes are heard; there is a prolonged expiratory phase. Heart
sounds are distant. Which of the following is the most appropriate next step in diagnosis?
(A) CT scan of the chest
(B) Echocardiography
(C) Exercise stress testing
(D) PPD skin testing and chest x-ray
(E) Pulmonary function testing
105. A 45-year-old man comes to the office because of a 6-month history of difficulty sleeping, nervousness, and fatigue. He goes to
bed at 11
PM every night, falls asleep within 15 minutes, then sleeps “fitfully.” After he awakens, he has mild neck pain and feels
tired, edgy, and tense during the rest of the day. The company where he worked for 27 years closed 6 months ago, and he has
been unable to find another job. He says he worries about his finances and job prospects. Medical history is remarkable for alcohol
use disorder. He has not drunk alcohol for 1 year. He says he is committed to being sober but recently has “wanted a drink.” He
currently takes no medications. Vital signs are within normal limits. Physical examination discloses no abnormalities. Mental
status examination shows a constricted, anxious affect. Which of the following is the most appropriate pharmacotherapy?
(A) Buspirone
(B) Clonazepam
(C) Diphenhydramine
(D) Imipramine
(E) Quetiapine
(F) Temazepam
106. A 56-year-old man comes to the clinic with his wife because of a 6-month history of restlessness while sleeping. The wife says
he motions as if he were throwing or kicking a ball and sometimes punches the pillow during the night; he has hit her twice.
Recently, he began talking in his sleep and yelling expletives. Occasionally, he seems to be “acting out” a dream; twice, he has
awakened and could tell his wife what the dream was about. Otherwise, he is unaware of these episodes and feels well rested in
the mornings. He has a history of constipation treated with polyethylene glycol. He takes no other medications. Vital signs are
within normal limits. Examination shows no abnormalities. This patient is at greatest risk for developing which of the following?
(A) Alzheimer disease
(B) Amyotrophic lateral sclerosis
(C) Cerebral infarction
(D) Narcolepsy
(E) Parkinson disease
58
107. A 77-year-old woman is brought to the emergency department by her son because of a 3-day history of pain in her right wrist.
The patient has advanced dementia, Alzheimer type, and is unable to provide her medical history. She is completely dependent
on her son, with whom she lives and who is her only caregiver. The son says he is unaware of any trauma to the patient's wrist.
The son says the patient has no other history of serious illness. Her only medication is memantine. She is nonverbal and has a flat
affect. She appears unkempt and to have poor hygiene. Temperature is 36.7°C (98.1°F), pulse is 82/min and regular, respirations
are 18/min, and blood pressure is 150/90 mm Hg. The patient moans when the physician touches her right wrist. Examination
shows multiple bruises over both forearms and wrists. The remainder of the examination shows no abnormalities. X-ray of the
right wrist shows a distal radial fracture; the fracture is treated with cast immobilization. Which of the following is the most
appropriate next step in management?
(A) Admit the patient to the hospital for further evaluation
(B) Arrange for placement of the patient in a nursing care facility
(C) Contact adult protective services
(D) Provide phone numbers of nearby nursing care facilities
(E) Recommend further evaluation at a clinic specializing in geriatric patients
108. A 27-year-old man comes to the emergency department because of a 4-week history of progressive shortness of breath and
swelling of the lower extremities. During this time, he also has had shortness of breath during the night while lying down. He
also has a 6-month history of intermittent, moderate substernal chest pain and pressure while exercising. He has not had syncope.
He has no history of serious illness and takes no medications. Temperature is 37.0°C (98.6°F), pulse is 110/min, respirations are
24/min, and blood pressure is 145/92 mm Hg. Examination shows severe, diffuse gingivitis. Crackles are heard halfway up the
posterior lung bases bilaterally. On cardiac examination, a prominent S
3
is heard. There is 3+ pitting edema of the lower
extremities. ECG shows sinus tachycardia. Which of the following is the most likely substance used?
(A) Cocaine
(B) Heroin
(C) Methadone
(D) Methamphetamine
(E) Toluene
109. A 39-year-old man comes to the office because of a 3-month history of episodes of headaches, palpitations, and sweating. The
episodes occur two to three times monthly. He has hypertension treated with a calcium-channel blocking agent and an ACE
inhibitor. Blood pressure is 166/112 mm Hg. Examination shows no other abnormalities. CT scan of the abdomen shows a 4-cm
left adrenal mass. Which of the following is the most appropriate next step in management?
(A) 24-Hour urine collection for measurement of vanillylmandelic acid and 5-hydroxyindoleacetic acid
(B) Adrenal venous sampling
(C) Laparoscopic left adrenalectomy and right adrenal biopsy
(D) Measurement of plasma and urine catecholamine and metabolite concentrations
(E) Transsphenoidal hypophysectomy
110. A 41-year-old woman is being prepared for discharge from the hospital after treatment for acute pancreatitis. She has tolerated
oral fluids well for the past 12 hours. She has had three prior admissions for pancreatitis. Medical history also is remarkable for
alcohol use disorder. The patient does not have secure housing. Which of the following recommendations is most likely to reduce
this patient's risk for future hospital admissions?
(A) Abstinence from alcoholic beverages
(B) Enrollment in a health insurance plan
(C) Nutritional counseling
(D) Placement in a women's shelter
59
111. An 8-year-old boy is brought to the urgent care center by his mother while on a family vacation in Florida because of a 3-day
history of an itchy, burning rash. The ambient temperature during the past 3 days has been 37.8°C (100.0°F). The mother does
not recall any recent exposures for her son, and none of her other children have a rash. The patient's medical history is
unremarkable. Examination of the patient's back shows superficial, clear, 1- to 2-mm vesicles that resemble water droplets. The
vesicles rupture easily on palpation. There is no surrounding erythema. It is most appropriate for the physician to recommend
which of the following for this patient?
(A) Antifungal cream
(B) Heat avoidance
(C) Oral antibiotic therapy
(D) Oral antiviral therapy
(E) Oral retinoid therapy
(F) Ultraviolet therapy
112. A 55-year-old woman comes to the office for a follow-up examination 3 months after sustaining an uncomplicated myocardial
infarction in the distribution of the left anterior descending artery. She has participated in an outpatient rehabilitation program
since discharge from the hospital 6 weeks ago, but her attendance in the program has become irregular during the past 4 weeks.
When questioned about this, she reports a 4-week history of depressed mood, inability to enjoy activities she previously found
pleasurable, loss of appetite, and insomnia. She also has hyperlipidemia and hypertension. Medications are aspirin, atorvastatin,
and metoprolol. She is 163 cm (5 ft 4 in) tall and weighs 68 kg (150 lb); BMI is 26 kg/m
2
. Vital signs are within normal limits.
Lungs are clear to auscultation. Cardiac examination discloses a normal rhythm. There is no peripheral edema. Results of serum
studies are within the reference ranges. Which of the following is the most likely effect of this patient's depression on her overall
health?
(A) It is likely to decrease her perceived quality of life but have no impact on her mortality
(B) It is likely to double her risk for death during the next 3 months
(C) It is likely to decrease her risk for ischemia because she is likely to rest and avoid socializing
(D) It likely will have no significant effect on her physical health
113. A 78-year-old man is evaluated in the hospital following admission 8 hours ago for treatment of aspiration pneumonia. He has a
5-year history of intermittent dysphagia resulting in spontaneous regurgitation of undigested food and liquids. Over-the-counter
antacids had provided no improvement in his dysphagia. Until the development of his recent fever, chest pain, and shortness of
breath, the patient had not had any other symptoms and had not sought medical care. He has no other history of serious illness.
In the hospital, he has been receiving intravenous piperacillin-tazobactam. He does not smoke cigarettes or drink alcoholic
beverages. Temperature is 38.1°C (100.6°F), pulse is 90/min, respirations are 24/min, and blood pressure is 100/65 mm Hg. He
appears comfortable and is sitting upright in bed. Decreased breath sounds are heard in the right lower lung field. Abdomen is
soft and nontender. Which of the following is the most likely underlying cause of this patient's dysphagia?
(A) Achalasia
(B) Esophageal cancer
(C) Hiatal hernia
(D) Zenker diverticulum
60
114. A 27-year-old woman comes to the office because of a 3-day history of constant mild pain in her left lower abdomen. Vigorous
movement makes the pain worse. She has not had nausea. She is otherwise healthy and takes no medications. Menses occur at
regular 28-day intervals and last 4 to 6 days. Her last menstrual period was 3 weeks ago. She is sexually active with one male
partner, and they use condoms inconsistently. Temperature is 36.7°C (98.0°F), pulse is 80/min, respirations are 18/min, and blood
pressure is 104/60 mm Hg. The abdomen is flat and soft with voluntary guarding of the left lower quadrant. Bowel sounds are
normal. Pelvic examination shows tender fullness of the left adnexa. Urine pregnancy test is negative. Ultrasonography shows a
normal uterus with a 3 × 4-cm left adnexal cyst and a small amount of free fluid in the pelvis. Which of the following is the most
appropriate next step in management?
(A) Antibiotic therapy
(B) Appendectomy
(C) CT scan-guided aspiration
(D) Laparoscopy
(E) Observation only
115. A 60-year-old woman comes to the office for a health maintenance examination. She says she feels well. She has no history of
serious illness and takes no medications. Vital signs are within normal limits. Observation of the tympanic membranes is limited
because of cerumen filling both external auditory canals. The external ears appear normal. No other abnormalities are noted. On
questioning, she says she hears well. Which of the following is the most appropriate next step in management?
(A) Irrigation of the ear canals by the physician
(B) Manual removal of the cerumen by an otorhinolaryngologist
(C) Recommendation for daily use of cotton swabs
(D) Recommendation for use of a ceruminolytic agent
(E) No further management is indicated at this time
116. A hospitalized 37-year-old woman has a 2-hour history of severe chest pain and blood-streaked sputum 4 days after undergoing
hysterectomy and oophorectomy for ovarian cancer. Current medications are oxycodone, prophylactic low-molecular-weight
heparin, and a multivitamin. Temperature is 37.0°C (98.6°F), pulse is 90/min, respirations are 22/min, and blood pressure is
110/70 mm Hg. Examination shows a left pleural rub. An x-ray and CT scan of the chest show a triangular pleural-based density
in the lower lobe of the left lung. No pulmonary emboli are seen in the major branches. Venous duplex ultrasonography of the
lower extremities shows no deep venous thrombosis. Which of the following is the most likely diagnosis?
(A) Air embolism
(B) Empyema
(C) Hemothorax
(D) Pericarditis
(E) Pneumonia
(F) Pulmonary hemorrhage
(G) Pulmonary hypertension
(H) Pulmonary infarction
61
117. A 77-year-old man is brought to the emergency department because of a 1-hour history of severe, constant abdominal pain and
dizziness. He has hypertension well controlled with hydrochlorothiazide. He has smoked two packs of cigarettes daily for 60
years. He is 175 cm (5 ft 9 in) tall and weighs 113 kg (250 lb); BMI is 37 kg/m
2
. Temperature is 37.0°C (98.6°F), pulse is 120/min,
respirations are 25/min, and blood pressure is 80/60 mm Hg. Pulse oximetry on room air shows an oxygen saturation of 93%. He
appears uncomfortable. Abdominal examination shows mild distention; bowel sounds are decreased. Groin pulses are 1+.
Dorsalis pedis pulses are not palpable. Hematocrit is 37% and leukocyte count is 10,000/mm
3
. Serum electrolyte concentrations,
liver function tests, and amylase activity are within the reference ranges. An x-ray of the abdomen is shown. Which of the
following is the most likely diagnosis?
(A) Acute pancreatitis
(B) Mesenteric ischemia
(C) Perforated duodenal ulcer
(D) Ruptured abdominal aortic aneurysm
(E) Small-bowel obstruction
118. A 1-week-old male newborn is evaluated in the hospital nursery 4 days after undergoing operative repair of a myelomeningocele.
He was born at 37 weeks' gestation via uncomplicated, spontaneous vaginal delivery to an 18-year-old primigravid woman. The
defect was diagnosed prenatally by ultrasonography, and the repair was done 72 hours after delivery. He is at the 5th percentile
for length, 10th percentile for weight, and 25th percentile for head circumference. Temperature is 36.9°C (98.4°F), pulse is
138/min, and respirations are 42/min. Pulse oximetry on room air shows an oxygen saturation of 98%. The newborn appears
comfortable. Physical examination shows a soft, flat, anterior fontanel. Red reflexes are normal. Moro reflex is symmetric in the
upper extremities; there is no movement in the lower extremities. Ultrasonography of the brain shows mildly enlarged ventricles.
Which of the following is the most likely associated finding in this patient?
(A) Congenital heart malformation
(B) Cryptorchidism
(C) Impaired folate metabolism
(D) Neurogenic bladder
(E) Severe cognitive impairment
62
119. A randomized controlled trial is conducted to assess the effectiveness of a new antiarrhythmic drug in patients with recurrent
ventricular tachycardia. A total of 100 participants with ventricular tachycardia are randomly assigned to receive either the new
drug or a placebo. Results show that the new drug group has fewer episodes of recurrent ventricular tachycardia compared with
the placebo group. The difference was found to be statistically significant (p<0.05). The investigators conclude that the new drug
can be used to decrease mortality from cardiac arrhythmia. Which of the following factors most likely invalidates the author's
conclusions?
(A) Extrapolation of findings beyond data
(B) Insufficient power
(C) No information regarding confidence interval
(D) Selection bias
120. A 42-year-old woman comes to the office because of a 2-week history of sore throat and fullness in her neck. She has no history
of serious illness and takes no medications. She appears anxious. Temperature is 37.8°C (100.0°F), pulse is 102/min, respirations
are 22/min, and blood pressure is 146/82 mm Hg. The oropharynx is clear, and there are no exudates. There is palpable fullness
and tenderness of the anterior aspect of the neck. There is no lymphadenopathy. Lungs are clear to auscultation. Results of a
complete blood count are within the reference ranges. Chest x-ray shows no abnormalities. Which of the following is the most
likely diagnosis?
(A) Acute mononucleosis
(B) Gastroesophageal reflux disease
(C) Laryngitis
(D) Subacute thyroiditis
(E) Tracheitis
63
Block 1 (Questions 140)
1.____
2.____
3.____
4.____
5.____
6.____
7.____
8.____
9.____
10.____
11.____
12.____
13.____
14.____
15.____
16.____
17.____
18.____
19.____
20.____
21.____
22.____
23.____
24.____
25.____
26.____
27.____
28.____
29.____
30.____
31.____
32.____
33.____
34.____
35.____
36.____
37.____
38.____
39.____
40.____
Block 2 (Questions 4180)
41.____
42.____
43.____
44.____
45.____
46.____
47.____
48.____
49.____
50.____
51.____
52.____
53.____
54.____
55.____
56.____
57.____
58.____
59.____
60.____
61.____
62.____
63.____
64.____
65.____
66.____
67.____
68.____
69.____
70.____
71.____
72.____
73.____
74.____
75.____
76.____
77.____
78.____
79.____
80.____
Block 3 (Questions 81120)
81.____
82.____
83.____
84.____
85.____
86.____
87.____
88.____
89.____
90.____
91.____
92.____
93.____
94.____
95.____
96.____
97.____
98.____
99.____
100.____
101.____
102.____
103.____
104.____
105.____
106.____
107.____
108.____
109.____
110.____
111.____
112.____
113.____
114.____
115.____
116.____
117.____
118.____
119.____
120.____
ANSWER FORM FOR USMLE STEP 2 CK SAMPLE TEST QUESTIONS
64
Block 1 (Questions 140)
1. A
2. E
3. A
4. A
5. E
6. C
7. B
8. B
9. B
10. F
11. B
12. B
13. B
14. D
15. C
16. B
17. E
18. E
19. C
20. B
21. C
22. E
23. C
24. B
25. D
26. A
27. C
28. B
29. A
30. B
31. B
32. A
33. B
34. A
35. C
36. E
37. D
38. D
39. C
40. A
Block 2 (Questions 4180)
41. D
42. A
43. C
44. E
45. B
46. C
47. A
48. A
49. C
50. C
51. C
52. A
53. F
54. E
55. B
56. E
57. D
58. F
59. D
60. D
61. D
62. E
63. B
64. C
65. E
66. A
67. C
68. D
69. B
70. C
71. C
72. C
73. A
74. E
75. E
76. D
77. C
78. B
79. B
80. D
Block 3 (Questions 81120)
81. D
82. D
83. E
84. A
85. A
86. B
87. B
88. E
89. E
90. A
91. D
92. B
93. D
94. E
95. C
96. E
97. B
98. C
99. A
100. C
101. C
102. B
103. D
104. E
105. A
106. E
107. C
108. D
109. D
110. A
111. B
112. B
113. D
114. E
115. E
116. H
117. D
118. D
119. A
120. D
Answer Key for USMLE Step 2 CK Sample Test Questions